What packages do people load by default in LaTeX?












401














I'm getting the impression from reading the answers written by some of the real experts here that there are quite a few little packages that just tweak LaTeX2e's default behaviour a little to make it more sensible here and there.



Rather than try to pick these up one by one as I read answers to questions (and thus risk missing them), I thought I'd ask up front what LaTeX2e packages people load by default in (almost) every document.



As this is a "big list" question, I'm making it CW. I don't know if there are standard rules across all SE/SO sites for such questions, but on MathOverflow the rule is generally: one thing (in this case, package) per answer. I guess that if a couple of packages really do go together then it would be fine to group them.



This is perhaps a little subjective and a little close to the line, so I'll not be offended if it gets closed or voted down! (But please explain why in the comments.)



Also see our community poll question: “I have used the following packages / classes”










share|improve this question
























  • There are standard rules across all SE sites, see meta.stackexchange.com/questions/11740/… and follow the links. The idea is that the answer to a "what are good default packages" question is way too big for a single user to write, so the community helps out. The one accepted answer that everyone edits has lots of edits from lots of people. Anton Geraschenko of MO made his own very different interpretation, "post one resource per answer" (mathoverflow.net/faq#communitywiki), and we'll have to decide one or the other.
    – Kevin Vermeer
    Jul 29 '10 at 22:25








  • 7




    Personally, I'd find a single list, separated by headings (Ex. Format, Math, Bib,Images, Other for this question), with a list of everyone's packages and how they're different from other packages in the section much more readable and useful. That amsmath is the highest voted just says that the MO community is here in full force. The less-known, but equally relevant formatting packages linked by Vivi, Joseph, and András are invisible without a lot of scrolling and reading.
    – Kevin Vermeer
    Jul 29 '10 at 22:37






  • 5




    I think the list of one package per answer is a good idea, as we can vote on individual packages...
    – Amir Rachum
    Jul 30 '10 at 11:30










  • My intention was not so much to find an ordering, but rather to find if there are any that I'd never heard of. It's not working out quite as I'd hoped, but I'm not sure if its possible to fix it at this stage (or worth doing).
    – Loop Space
    Jul 30 '10 at 11:37






  • 1




    It can be good to have a single answer that is just an index of all the other answers, and accept that, so that it floats to the top.
    – naught101
    Aug 30 '12 at 3:44
















401














I'm getting the impression from reading the answers written by some of the real experts here that there are quite a few little packages that just tweak LaTeX2e's default behaviour a little to make it more sensible here and there.



Rather than try to pick these up one by one as I read answers to questions (and thus risk missing them), I thought I'd ask up front what LaTeX2e packages people load by default in (almost) every document.



As this is a "big list" question, I'm making it CW. I don't know if there are standard rules across all SE/SO sites for such questions, but on MathOverflow the rule is generally: one thing (in this case, package) per answer. I guess that if a couple of packages really do go together then it would be fine to group them.



This is perhaps a little subjective and a little close to the line, so I'll not be offended if it gets closed or voted down! (But please explain why in the comments.)



Also see our community poll question: “I have used the following packages / classes”










share|improve this question
























  • There are standard rules across all SE sites, see meta.stackexchange.com/questions/11740/… and follow the links. The idea is that the answer to a "what are good default packages" question is way too big for a single user to write, so the community helps out. The one accepted answer that everyone edits has lots of edits from lots of people. Anton Geraschenko of MO made his own very different interpretation, "post one resource per answer" (mathoverflow.net/faq#communitywiki), and we'll have to decide one or the other.
    – Kevin Vermeer
    Jul 29 '10 at 22:25








  • 7




    Personally, I'd find a single list, separated by headings (Ex. Format, Math, Bib,Images, Other for this question), with a list of everyone's packages and how they're different from other packages in the section much more readable and useful. That amsmath is the highest voted just says that the MO community is here in full force. The less-known, but equally relevant formatting packages linked by Vivi, Joseph, and András are invisible without a lot of scrolling and reading.
    – Kevin Vermeer
    Jul 29 '10 at 22:37






  • 5




    I think the list of one package per answer is a good idea, as we can vote on individual packages...
    – Amir Rachum
    Jul 30 '10 at 11:30










  • My intention was not so much to find an ordering, but rather to find if there are any that I'd never heard of. It's not working out quite as I'd hoped, but I'm not sure if its possible to fix it at this stage (or worth doing).
    – Loop Space
    Jul 30 '10 at 11:37






  • 1




    It can be good to have a single answer that is just an index of all the other answers, and accept that, so that it floats to the top.
    – naught101
    Aug 30 '12 at 3:44














401












401








401


433





I'm getting the impression from reading the answers written by some of the real experts here that there are quite a few little packages that just tweak LaTeX2e's default behaviour a little to make it more sensible here and there.



Rather than try to pick these up one by one as I read answers to questions (and thus risk missing them), I thought I'd ask up front what LaTeX2e packages people load by default in (almost) every document.



As this is a "big list" question, I'm making it CW. I don't know if there are standard rules across all SE/SO sites for such questions, but on MathOverflow the rule is generally: one thing (in this case, package) per answer. I guess that if a couple of packages really do go together then it would be fine to group them.



This is perhaps a little subjective and a little close to the line, so I'll not be offended if it gets closed or voted down! (But please explain why in the comments.)



Also see our community poll question: “I have used the following packages / classes”










share|improve this question















I'm getting the impression from reading the answers written by some of the real experts here that there are quite a few little packages that just tweak LaTeX2e's default behaviour a little to make it more sensible here and there.



Rather than try to pick these up one by one as I read answers to questions (and thus risk missing them), I thought I'd ask up front what LaTeX2e packages people load by default in (almost) every document.



As this is a "big list" question, I'm making it CW. I don't know if there are standard rules across all SE/SO sites for such questions, but on MathOverflow the rule is generally: one thing (in this case, package) per answer. I guess that if a couple of packages really do go together then it would be fine to group them.



This is perhaps a little subjective and a little close to the line, so I'll not be offended if it gets closed or voted down! (But please explain why in the comments.)



Also see our community poll question: “I have used the following packages / classes”







packages big-list






share|improve this question















share|improve this question













share|improve this question




share|improve this question








edited Mar 16 '17 at 16:37


























community wiki





4 revs, 3 users 48%
Loop Space













  • There are standard rules across all SE sites, see meta.stackexchange.com/questions/11740/… and follow the links. The idea is that the answer to a "what are good default packages" question is way too big for a single user to write, so the community helps out. The one accepted answer that everyone edits has lots of edits from lots of people. Anton Geraschenko of MO made his own very different interpretation, "post one resource per answer" (mathoverflow.net/faq#communitywiki), and we'll have to decide one or the other.
    – Kevin Vermeer
    Jul 29 '10 at 22:25








  • 7




    Personally, I'd find a single list, separated by headings (Ex. Format, Math, Bib,Images, Other for this question), with a list of everyone's packages and how they're different from other packages in the section much more readable and useful. That amsmath is the highest voted just says that the MO community is here in full force. The less-known, but equally relevant formatting packages linked by Vivi, Joseph, and András are invisible without a lot of scrolling and reading.
    – Kevin Vermeer
    Jul 29 '10 at 22:37






  • 5




    I think the list of one package per answer is a good idea, as we can vote on individual packages...
    – Amir Rachum
    Jul 30 '10 at 11:30










  • My intention was not so much to find an ordering, but rather to find if there are any that I'd never heard of. It's not working out quite as I'd hoped, but I'm not sure if its possible to fix it at this stage (or worth doing).
    – Loop Space
    Jul 30 '10 at 11:37






  • 1




    It can be good to have a single answer that is just an index of all the other answers, and accept that, so that it floats to the top.
    – naught101
    Aug 30 '12 at 3:44


















  • There are standard rules across all SE sites, see meta.stackexchange.com/questions/11740/… and follow the links. The idea is that the answer to a "what are good default packages" question is way too big for a single user to write, so the community helps out. The one accepted answer that everyone edits has lots of edits from lots of people. Anton Geraschenko of MO made his own very different interpretation, "post one resource per answer" (mathoverflow.net/faq#communitywiki), and we'll have to decide one or the other.
    – Kevin Vermeer
    Jul 29 '10 at 22:25








  • 7




    Personally, I'd find a single list, separated by headings (Ex. Format, Math, Bib,Images, Other for this question), with a list of everyone's packages and how they're different from other packages in the section much more readable and useful. That amsmath is the highest voted just says that the MO community is here in full force. The less-known, but equally relevant formatting packages linked by Vivi, Joseph, and András are invisible without a lot of scrolling and reading.
    – Kevin Vermeer
    Jul 29 '10 at 22:37






  • 5




    I think the list of one package per answer is a good idea, as we can vote on individual packages...
    – Amir Rachum
    Jul 30 '10 at 11:30










  • My intention was not so much to find an ordering, but rather to find if there are any that I'd never heard of. It's not working out quite as I'd hoped, but I'm not sure if its possible to fix it at this stage (or worth doing).
    – Loop Space
    Jul 30 '10 at 11:37






  • 1




    It can be good to have a single answer that is just an index of all the other answers, and accept that, so that it floats to the top.
    – naught101
    Aug 30 '12 at 3:44
















There are standard rules across all SE sites, see meta.stackexchange.com/questions/11740/… and follow the links. The idea is that the answer to a "what are good default packages" question is way too big for a single user to write, so the community helps out. The one accepted answer that everyone edits has lots of edits from lots of people. Anton Geraschenko of MO made his own very different interpretation, "post one resource per answer" (mathoverflow.net/faq#communitywiki), and we'll have to decide one or the other.
– Kevin Vermeer
Jul 29 '10 at 22:25






There are standard rules across all SE sites, see meta.stackexchange.com/questions/11740/… and follow the links. The idea is that the answer to a "what are good default packages" question is way too big for a single user to write, so the community helps out. The one accepted answer that everyone edits has lots of edits from lots of people. Anton Geraschenko of MO made his own very different interpretation, "post one resource per answer" (mathoverflow.net/faq#communitywiki), and we'll have to decide one or the other.
– Kevin Vermeer
Jul 29 '10 at 22:25






7




7




Personally, I'd find a single list, separated by headings (Ex. Format, Math, Bib,Images, Other for this question), with a list of everyone's packages and how they're different from other packages in the section much more readable and useful. That amsmath is the highest voted just says that the MO community is here in full force. The less-known, but equally relevant formatting packages linked by Vivi, Joseph, and András are invisible without a lot of scrolling and reading.
– Kevin Vermeer
Jul 29 '10 at 22:37




Personally, I'd find a single list, separated by headings (Ex. Format, Math, Bib,Images, Other for this question), with a list of everyone's packages and how they're different from other packages in the section much more readable and useful. That amsmath is the highest voted just says that the MO community is here in full force. The less-known, but equally relevant formatting packages linked by Vivi, Joseph, and András are invisible without a lot of scrolling and reading.
– Kevin Vermeer
Jul 29 '10 at 22:37




5




5




I think the list of one package per answer is a good idea, as we can vote on individual packages...
– Amir Rachum
Jul 30 '10 at 11:30




I think the list of one package per answer is a good idea, as we can vote on individual packages...
– Amir Rachum
Jul 30 '10 at 11:30












My intention was not so much to find an ordering, but rather to find if there are any that I'd never heard of. It's not working out quite as I'd hoped, but I'm not sure if its possible to fix it at this stage (or worth doing).
– Loop Space
Jul 30 '10 at 11:37




My intention was not so much to find an ordering, but rather to find if there are any that I'd never heard of. It's not working out quite as I'd hoped, but I'm not sure if its possible to fix it at this stage (or worth doing).
– Loop Space
Jul 30 '10 at 11:37




1




1




It can be good to have a single answer that is just an index of all the other answers, and accept that, so that it floats to the top.
– naught101
Aug 30 '12 at 3:44




It can be good to have a single answer that is just an index of all the other answers, and accept that, so that it floats to the top.
– naught101
Aug 30 '12 at 3:44










63 Answers
63






active

oldest

votes













1 2
3
next












246














I almost always load microtype. It plays with ever-so-slightly shrinking and stretching of the fonts and with the extent to which text protrudes into the margins in a way that yields results that look better, that have fewer instances of hyphenation, and fewer overfull hboxes. It doesn't work with latex, you have to use pdflatex instead. It also works with lualatex and (protrusion only) with xelatex.






share|improve this answer



















  • 14




    You may want to use usepackage[stretch=10]{microtype}, which allows font expansion up to 1% (default is 2%).
    – lockstep
    Aug 6 '10 at 12:03






  • 69




    Can we have an example of with versus without?
    – levesque
    Nov 15 '10 at 18:28






  • 9




    there's a nice example in the documentation for microtype mirror.ctan.org/macros/latex/contrib/microtype/microtype.pdf, though it requires adobe acrobat for the inline examples
    – Noah
    Aug 12 '11 at 22:37






  • 11




    Here is another example.
    – Juri Robl
    Oct 11 '12 at 11:13






  • 4




    Here is a superb page of examples with and without microtype: khirevich.com/latex/microtype
    – Travis Bemrose
    Aug 19 '16 at 4:03



















199














The family of AMS math packages. At least amsmath and amssymb. Also amsthm if I need theorems and the class I'm using doesn't already define them.



Particularly for writing equations, the AMS packages define a rich set of environments to group and align formulas in many different and useful ways. I also like that it encourages the use of semantic commands (e.g. the cases environment) over syntactic commands (e.g. a left{ followed by an array).



Its documentation can be found running texdoc amsldoc on a command line.






share|improve this answer



















  • 5




    In particular, amsthm provides an easy way to set up different theorem styles, amsmath provides the text command, and amssymb contains several often-used symbols.
    – András Salamon
    Jul 29 '10 at 12:40






  • 22




    +1 for the (oblique) reference to texdoc. I only discovered that recently and I wonder how I ever lived without it!
    – Loop Space
    Jul 29 '10 at 18:08






  • 6




    I believe amssymb loads amsfonts. There's rarely any need to load it yourself.
    – TH.
    Sep 11 '10 at 9:13






  • 5




    Note that the ams math packages are loaded automatically if you use one of their document classes, such as amsart.
    – Erik P.
    Jan 18 '12 at 19:08






  • 12




    Instead of loading amsmath I usually load mathtools. It is based on amsmath and loads it automatically. Moreover it fixes some deficiencies of the amsmath package and provides additional useful commands such as coloneqq.
    – Stan
    Aug 24 '14 at 9:53





















171














I use hyperref for setting PDF metadata and to create links, both within the document and for clickable URLs. Even Elsevier has used urlbst to update their bibliography style to support URLs and DOIs; hyperref does the actual work of rendering url = and doi = BibTeX fields into clickable PDF links.






share|improve this answer























  • related question: tex.stackexchange.com/questions/1863/…
    – matth
    Jun 26 '12 at 21:25



















155














For citations and bibliographies, biblatex is the package of my choice. Key points:




  • biblatex includes a wide variety of built-in citation/bibliography styles (numeric, alphabetic, author-year, author-title, verbose [full in-text-citations], with numerous variants for each one). A number of custom styles have been published.


  • Modifications of the built-in or custom styles can be accomplished using LaTeX macros instead of having to resort to the BibTeX programming language.


  • biblatex offers well-nigh every feature of other bibliography-related LaTeX packages (e.g. multiple/subdivided bibliographies, sorted/compressed citations, entry sets, ibidem functionality, back references). If a feature is not included, chances are high it is on the package authors' to-do list.


  • The babel package is supported, and biblatex comes with localization files for about a dozen languages (with the list still growing).


  • Although the current version of biblatex (2.8a) still allows to use BibTeX as a database backend, by default it cooperates with Biber which supports bibliographies using Unicode. Biber (currently at version 1.8) is included in TeX Live and MiKTeX. Many features introduced since biblatex 1.1 (e.g., advanced name disambiguation, smart crossref data inheritance, configurable sorting schemes, dynamic datasource modification) are "Biber only".







share|improve this answer



















  • 7




    Nevertheless one should append about the usage of biblatex that some papers do not accept its usage. See: Biblatex: submitting to a journal
    – strpeter
    Jan 16 '14 at 9:25





















138














The todonotes package is a must have in all my documents.



usepackage{todonotes}


The package enables you to insert small notes in the text marking things to do in the document. Something like



todo{Rewrite this answer ldots}


At any location in the document a list of the inserted notes can be generated with the



listoftodos 


command.






share|improve this answer



















  • 11




    For multiuser comment support, and configurability with regard to the kinds of notes/themes available, the fixme package is quite nice (I use it quite regularly).
    – Mark
    Mar 25 '11 at 22:29










  • todonotes also supports colors and missing graphics.
    – ℝaphink
    Jun 16 '11 at 10:42










  • I find todonote invaluable as I prepare syllabai and course material for the upcoming term. Because I cannot do everything at one sitting I put a todo note whenever i find something I have to wait to do. I also use it during the semester to highlight to the students anything which was changed after the documents were first published.
    – R. Schumacher
    May 1 '12 at 20:51








  • 4




    Personally I use an editor which automatically highlights and groups in the "structure" window any comment that begins with %TODO: Works better for me because you don't have anything in your compiled document giving away the fact that it still has TODOs around.
    – Dom
    Jun 12 '13 at 10:53






  • 12




    Has anyone done a comparison between easy-todo, fixme, fixmetodonotes, todo, and todonotes?
    – Ari Brodsky
    Nov 20 '13 at 1:12



















132














One package that’s really general purpose is nag: It doesn’t do anything, per se, it just warns when you accidentally use deprecated LaTeX constructs from l2tabu (English / French / German / Italian / Spanish documentation).



From the documentation:




Old habits die hard. All the same, there are commands, classes and packages which are outdated and superseded. nag provides routines to warn the user about the use of those. As an example, we provide an extension that detects many of the “sins” described in l2tabu.




Therefore, I now always have the following in my header (before the documentclass, thanks qbi):



RequirePackage[l2tabu, orthodox]{nag}


It’s a bit like having use strict; in Perl: a useful best practice.






share|improve this answer



















  • 22




    Somewhat better is RequirePackage[l2tabu,orthodox]{nag} before documentclass. The package docu also recommends this.
    – qbi
    Jul 29 '10 at 18:40






  • 5




    This package sounds useful. However, when I tested it with a large project, I started to get the message "Label(s) may have changed. Rerun to get cross-references right." no matter how many times I re-run Latex.
    – Jukka Suomela
    Jul 31 '10 at 9:36



















114














I'm surprised that no one has mentioned



usepackage[margin=1in]{geometry} % set page margins automatically 


This is in every document I write (with varying margins, of course.)






share|improve this answer



















  • 13




    This is generally poor style. The design of the page is pretty involved and lots of thought has went into (La)TeX's default designs. If you're interested in just saving paper, consider the packages savetrees or fullpage.
    – Quadrescence
    Apr 16 '11 at 23:15








  • 23




    Both savetrees and fullpage change other things too; Anyway, the point of of the answer is that geometry is a must use package, no matter what margins you choose for it. The appropriateness of 1in margins also depends on the kind of documents you produce.
    – Alan Munn
    Apr 16 '11 at 23:38






  • 12




    It is not a must if you use a class from the KOMAscript bundle or memoir.
    – Sveinung
    Jan 13 '14 at 16:01










  • @Sveinung but it is a must if you have to comply with certain margins, because your supervisor/advisor/publisher orders you to do so.
    – Skillmon
    Dec 14 '18 at 20:08










  • @Skillmon No, because memoir and KOMA have their own ways to set margins, so geometry isn't needed, which is what Sveinung means, I suppose.
    – Alan Munn
    Dec 14 '18 at 20:10





















114














Another essential package combination is



usepackage{booktabs}
usepackage{array}


The booktabs package creates much nicer looking tables than the standard latex tables; the array package's ability to create custom columns is invaluable for formatting tabular material on a per-column basis.






share|improve this answer



















  • 1




    I just discovered booktabs -- it is great!
    – Ben
    Jan 12 '11 at 22:37






  • 1




    @Ben Yes, it's a great package. If you visit my profile web link you can find my own list of essential packages.
    – Alan Munn
    Jan 12 '11 at 22:47



















113














I nearly always use the tikz package. Once you learn how to draw with it, you can do almost any vector graphic you need.






share|improve this answer























  • I have always used Inkscape for the production of my vector images, diagrams and whatsoever. Does tikz produce comparable diagrams? How much effort is involved?
    – levesque
    Nov 15 '10 at 18:28






  • 4




    You can produce almost any diagram with Tikz. Check the tikz examples page. texample.net/tikz/examples However, it is fairly complicated to get the hang on large diagrams since you have to type everything and nearly always you can't see what you are doing. But if you are using a Debian/KDE combination, you can use Ktikz/Qtikz which is really helpful since it compiles tikz code in real time.
    – fabikw
    Nov 16 '10 at 0:42








  • 35




    TikZ is awesome with a capital A. But load it by default? It takes up a lot of time and space. I would say only load it if you need it.
    – Matthew Leingang
    Nov 22 '10 at 12:53










  • It takes time, but nearly always I find I need to do something with it.
    – fabikw
    Nov 23 '10 at 1:11






  • 9




    @levesque: Tikz has a fairly steep learning curve, but it is beautifully documented and provides rich libraries. I find the vector graphics that I produce in tikz to be superior to those I produced in inkscape. It seems easier on my brain to stay in keyboard mode as well.
    – philosodad
    Dec 29 '10 at 4:56



















107














Since my files nowadays has UTF-8 character encoding, I use this



usepackage[utf8]{inputenc}





share|improve this answer



















  • 37




    XeLaTeX or LuaLaTeX would be my choice for this
    – Joseph Wright
    Aug 15 '10 at 13:05










  • Interesting, must look into those projects.
    – Johan
    Aug 15 '10 at 13:16






  • 1




    Isn't it usepackage[utf8x]{inputenc}?
    – Olivier
    Jul 19 '11 at 8:17






  • 1




    I've experienced several cases where utf8x had a symbol that utf8 hadn't
    – Mog
    Nov 24 '12 at 11:47






  • 8




    @Olivier: utf8 is LaTeX base, while utf8x comes from the ucs package. So utf8 is portable.
    – Martin Schröder
    Jun 27 '13 at 14:39



















103














usepackage{siunitx}


siunitx, for typesetting units and especially for the "S" column type, which allows numbers in tables to be easily aligned, e.g. on the decimal marker.






share|improve this answer



















  • 12




    usepackage[allowlitunits]{siunitx} is my normal incantation, it allows you to use things like 20millimeter directly in math mode.
    – Alex Hirzel
    May 1 '12 at 20:18












  • I have evidently never followed through on my plan to read the siunitx manual in depth. I was not aware of the S column type or allowlitunits, thank you!
    – owjburnham
    Jul 18 '17 at 9:50



















79














The 'rich' document classes such as memoir and KOMA-Script include a lot of functionality that is not available from the LaTeX kernel. So the packages you load when using the article class might be rather different from those when using memoir. A lot of packages that get used by many people with the base classes (things like float, caption, tocbibind and titlesec) are covered by the richer document classes.






share|improve this answer



















  • 16




    begin{gripe} My problems with these richer document classes are that it makes it very difficult to pick and choose, and that it is a major pain when Big Shot Journal says "please rewrite your document to use our class file" (there's even a journal that won't let you send an accompanying style file). end{gripe}
    – Loop Space
    Jul 29 '10 at 13:19






  • 10




    I tend to stick to article + packages, myself, so I can sympathise. All the more reason for me to get on and get LaTeX3 finished, so we can have a good set of abilities out of the box!
    – Joseph Wright
    Jul 29 '10 at 14:33






  • 9




    begin{joke} Then stop wasting time here and get on with it! end{joke}
    – Loop Space
    Jul 29 '10 at 18:11








  • 3




    If only it were that easy :-) If you want to see that things are happening, there is an RSS feed for SVN checkins: latex-project.org/latex3svn.rss
    – Joseph Wright
    Jul 29 '10 at 21:36






  • 13




    That gripe seems a gripe with the journals, rather than with the rich document classes. Also, if you're writing a journal article, memoir doesn't seem like an obvious way to go, if you are going to end up having to conform to some journal's style eventually. Again, that's not an issue with rich document classes, that's just a matter of picking the right tool for the job. And for journal submissions, minimal package requirements and basic document classes seems a good modus operandi
    – Seamus
    Aug 1 '10 at 10:41



















72














usepackage{graphicx}


For including figures, rotating or scaling text. I also use the graphicspath command to specify a subfolder to help organize my figures and so I can easily change between, for example, a set of figures for internal used (with extra info) and final versions for distribution.






share|improve this answer































    63














    usepackage{lmodern}  % better i18n Postscript version of Knuth's cm fonts





    share|improve this answer























    • related question: tex.stackexchange.com/questions/1390/latin-modern-vs-cm-super
      – matth
      Jun 26 '12 at 21:24






    • 1




      I never use this except for debugging. usepackage{cfr-lm} ...
      – cfr
      Nov 24 '16 at 0:58



















    61














    In addition to many packages already listed here, I always include mathtools. It provides implementations of mathclap (and similar commands) as well as nice extensible arrow.






    share|improve this answer



















    • 4




      mathclap is great. I use it to great effect for things like sum_{mathclap{big long thing}}. (It's also amusingly named with at least one off-color meaning.)
      – TH.
      Aug 27 '10 at 9:36






    • 2




      shortintertext is also provided by the mathtools package and provids tighter vertical spacing compared to intertext from the amsmath package.
      – Peter Grill
      May 2 '12 at 0:47



















    60














    I can't live without listings --- pretty-printing (colours, formatting and all) algorithms and code is indispensable --- in pretty much any programming languages and dialects under the sun. Plus, I can import a source file directly from the repository, and the latest version will be automatically rendered.






    share|improve this answer



















    • 1




      I was pleasantly surprised that I could prettyprint MIPS assembly language code with listings! Excellent package.
      – MercurialMadnessMan
      Nov 24 '12 at 6:30



















    56














    The package xspace lets you define commands that don't eat up whitespace after them. So you can define an abbreviation like



    newcommand{sA}{mathcal{A}xspace}



    and then you can type objects of sA are called widgets instead of objects of sA are called widgets.






    share|improve this answer



















    • 1




      That's one I use so much that I forget it's not part of the main code!
      – Loop Space
      Aug 5 '10 at 7:10






    • 20




      On comp.text.tex there's a series of messages "xspace and italic correction" about spacing inconsistencies created by xspace. There, Will Robertson suggested "delimited macros" as an alternative to xspace. Using newcommand* only to ensure that no existing command is overriden, the above example would look like this: newcommand*{sA}{}defsA/{mathcal{A}} To quote Will Robertson: "In the source you must always type "foo/" [here: "sA/"] (or TeX will throw an error), and spaces after it won't be gobbled."
      – lockstep
      Aug 6 '10 at 15:04






    • 6




      The main advantage of sa/ is that an error message will occur if you happen to forget the closing slash. On the contrary, if you happen to forget the closing backslash of sA , you'll end with gobbled space without noticing it.
      – lockstep
      Aug 11 '10 at 20:50








    • 6




      I used xspace one time in a paper with other authors. It was a huge pain since some macros didn't behave like others. It led to all sort of confusion, especially when thinks like foo bar no long work as you expect because foo's definition ends with xspace. I've never tried foo/. The main advantage I see with that is if your macro is m/...
      – TH.
      Aug 27 '10 at 9:32






    • 9




      I don't especially like the look of sA/ but I can't think of a better delimiter to use. Perhaps a semicolon would be fine (after HTML): sA;. My personal belief is that non-delimited macros without arguments (i.e., the ones that gobble spaces) are just plain wrong for document commands because of the spacing problems. Even experienced LaTeX authors trip up with them.
      – Will Robertson
      Sep 2 '10 at 9:28



















    52














    First line of the document should be



    RequirePackage{fixltx2e}
    documentclass{...}


    , which fixes a few things in the LaTeX2e kernel.



    Due to LaTeX's stability policy, these corrections have not been incorporated into the LaTeX2e kernel, but this package does things most people would agree are bugfixes. So to load this package is always recommended for newly created documents. The corrections have no commonalities, but the package's description has a nice summary:





    • ensure one-column floats don't get ahead of two-column floats;

    • correct page headers in twocolumn documents;

    • stop spaces disappearing in moving arguments;

    • allowing fnsymbol to use text symbols;

    • allow the first word after a float to hyphenate;


    • emph can produce caps/small caps text;

    • bugs in setlength and flushbottom.






    EDIT 27.01.2016:



    This package is obsolete for LaTeX releases after 2015. See latexrelease.pdf.






    share|improve this answer



















    • 1




      It should be RequirePackage{fixltx2e} as first line of you'require document, even before the document class, see texdev.net/2014/12/28/fixing-latex2e
      – MaxNoe
      Jan 17 '15 at 13:51






    • 1




      really should be an argument to documentclass.
      – ivo Welch
      Jan 2 '16 at 16:58






    • 7




      fixltx2e is not required with releases after 2015(fixltx2e) All fixes are now in the LaTeX kernel.
      – kaka
      Apr 3 '16 at 11:23



















    51














    For papers on the arXiv (maths, physics and computer science mostly) there's a list of packages sorted by frequency of use.



    The top twenty packages are:




    1. article

    2. graphicx

    3. amssymb

    4. amsmath

    5. revtex

    6. revtex4

    7. epsfig

    8. amsfonts

    9. bm

    10. latexsym

    11. amsart

    12. dcolumn

    13. amsthm

    14. graphics

    15. aastex

    16. amscd

    17. epsf

    18. color

    19. aa

    20. times






    share|improve this answer



















    • 28




      That list is literally pain to my eyes. Loading bm?! Use proper bold math characters instead, please, and not poorman's bold. times? Outdated since ages, use mathptmx or XITS Math instead. I'll stop here...
      – Ingo
      Jan 30 '14 at 11:46






    • 1




      @Ingo arXiv has been created in 1991 and some papers haven't been updated since then!
      – Najib Idrissi
      Feb 24 '17 at 14:57



















    44














    I use url to typeset urls.






    share|improve this answer































      32














      For quickly setting multicolumn text in a single column document, the multicol package is another package that I use all the time.



      usepackage{multicol}





      share|improve this answer































        29














        To use the palatino font (it's just a nice looking font)



        usepackage[sc]{mathpazo}


        Note that the old palatino package is deprecated.






        share|improve this answer























        • never use usepackage{palatino}, see l2tabu. the current way to use Palatino is usepackage{mathpazo}
          – Mateus Araújo
          Sep 2 '10 at 3:47










        • What is l2tabu?
          – Johan
          Sep 2 '10 at 6:35






        • 13




          You should probably also load mathpazo with the [sc] option to get real small caps and better kerning.
          – Will Robertson
          Sep 2 '10 at 9:24






        • 3




          Depending on taste, you may want to use [osf] instead of [sc] to get old style numerals as well as the real small caps and better kerning. I for one find old style numerals prettier and classier than lining figures in text mode (using [osf] will keep lining figures in math mode).
          – spet
          May 29 '13 at 8:52






        • 1




          According to this the LaTeX font catalogue, one should increase the leading when using mathpazo. tug.dk/FontCatalogue/urwpalladio
          – Ubiquitous
          Nov 21 '14 at 9:04



















        29














        usepackage[parfill]{parskip} 


        I much prefer no indentation and space between paragraphs, so the parskip package is a must for me!






        share|improve this answer



















        • 17




          Have a look at the KOMA-Script-classes - they include a parskip option that is more powerful than the package of the same name.
          – lockstep
          Aug 8 '10 at 17:39



















        27














        I almost always find myself using a tabularx environment as opposed to the regular tabular environment, as it allows for greater dynamism in column widths.






        share|improve this answer































          27














          Nothing surprising here: I use natbib, hyperref and hypernat together.



          Natbib for referencing.



          Hyperref adds bookmarks for sections and lists and turns references and urls into links.



          Hypernat allows natbib and hyperref to work together. -- Note (added 2015/02/11): natbib and hyperref have been working together just fine for at least ten years. hypernat is no longer needed for any TeX distribution with a vintage more recent than ca 2002.






          share|improve this answer



















          • 8




            I'm pretty sure that hypernat is superfluous these days. With only loading natbib and hyperref I get references as [1-5] with both 1 and 5 being hyperlinks.
            – Lev Bishop
            Aug 8 '10 at 14:51










          • Agreed, I didn't even know about hypernat until I saw this answer. I have been using hyperref and natbib for a while and reference links and backlinks always worked for me. Is there some extra functionality that hypernat adds?
            – Sharpie
            Aug 9 '10 at 17:31










          • I had a problem once, found out about natbib, and have been using it ever since, so it is possible it is superfluous and I didn't even know. I will have to test it out and get back to you guys if I find something.
            – Vivi
            Aug 10 '10 at 20:18






          • 2




            And? Was it superfluous in 2010? Is it now? ;)
            – K.-Michael Aye
            Nov 23 '12 at 5:18






          • 1




            @K.-MichaelAye - hypernat was superfluous (and potentially troublesome) back in 2010 and in 2012, and it continues to be superfluous as of 2015.
            – Mico
            Feb 11 '15 at 21:13



















          25














          I almost always use the enumitem package, which makes it much easier to make modifications to lists (especially enumerate lists). Most notably, changing the labels to something like (i), (ii), (iii) [no period] with this package is as easy as



          begin{enumerate}[label=(roman*)]
          item The first item
          item The second item
          end{enumerate}


          Furthermore, the code above will automatically get nesting right. Before I started using this package, my preamble always included the awkward macro (necessary to change the references and eliminate the extra period in the list itself)



          newcommand{setenumroman}{%
          renewcommand{theenumi}{(roman{enumi})}%
          renewcommand{labelenumi}{theenumi}%
          }


          which would break if I ever used it for a nested list (all the enumis would have to be changed to enumiis, if I understand correctly).



          The enumitem package is quite flexible; another option I sometimes use is [wide], which makes a list look like part of the body of the text (with numbers/labels at the beginning of relevant paragraphs).






          share|improve this answer























          • If someone only want the feature of changing labels, easier will be to use the enumerate package. Then you could simply write begin{enumerate}[(i)]. But enumitem package gives a lot more flexibility including allowing the items to appear in a line.
            – Cyriac Antony
            Dec 21 '18 at 8:38





















          24














          To make sure you have ISO formated dates (YYYY-MM-DD).



          usepackage[english]{isodate}


          or



          usepackage{datetime}
          renewcommand{dateseparator}{-}
          newcommand{todayiso}{theyear dateseparator twodigitmonth dateseparator twodigitday}





          share|improve this answer































            20














            Another package I use is float. It allows for the placement H for floats, which is somewhat equivalent to h!, but a bit stronger, making sure the figure or table goes exactly where I want it to be.






            share|improve this answer



















            • 6




              Actually not equivalent to h! at all. h! floats still "float"- they can be moved around by LaTeX in an attempt to optimize the document layout. Figures using the H specifier are not floats at all, they are treated like one big character and are put exactly where they appear in the text.
              – Sharpie
              Aug 1 '10 at 3:59












            • @Sharpie: you are ignoring the word "somewhat" :P Still, your point is valid, thanks!
              – Vivi
              Aug 1 '10 at 4:21






            • 1




              I did consider the word somewhat. However, in my opinion the only similarity between the two is the fact that they are used as float specifiers. Beyond that, both specifiers produce completely different effects.
              – Sharpie
              Aug 1 '10 at 6:14










            • @Sharpie: maybe I should link to the source of the (mis)information? en.wikibooks.org/wiki/LaTeX/… (see the last row of the table)
              – Vivi
              Aug 1 '10 at 6:32










            • @Vivi I fixed that entry of the wikibook.
              – Skillmon
              Dec 15 '18 at 10:46



















            19














            For mathematical texts I instead use amsmath & Co. One very useful package is onlyamsmath. I load it as



            usepackage[all,warning]{onlyamsmath}


            So it looks for $$..$$, eqnarray and produces a warning if some of them are used. If you left out warning, it will result in an error and compile will stop. This package is normally very useful if you edit a text with many authors.






            share|improve this answer































              19














              Edited by doncherry: Removed packages mentioned in separate answers.



              The complete header Part of my header for most of my documents looks as follows:



              documentclass[ngerman,draft,parskip=half*,twoside]{scrreprt}
              usepackage{ifthen}


              For some things I need if-then-constructs. This package provides an easy way to realise it.



              usepackage{index}


              For generating an index.



              usepackage{xcolor}


              xcolor is needed by several packages. For some historical reason I load it manually.



              usepackage{babel}
              usepackage{nicefrac}


              nicefrac allows typesetting fractions like 1/2. It is sometimes more readable than frac.



              usepackage[T1]{fontenc}
              usepackage[intlimits,leqno]{amsmath}
              usepackage[all,warning]{onlyamsmath}


              This package warns if non-amsmath-environments are used.



              usepackage{amssymb}
              usepackage{fixmath}


              Provides ISO conform greek letters.



              usepackage[euro]{isonums}


              Defines comma as decimal delimiter.



              usepackage[amsmath,thmmarks,hyperref]{ntheorem}


              for Theorems, definitions and stuff.



              usepackage{paralist}


              Improves enumerate and itemize. Also provides some compact environments.



              usepackage{svn}


              I work with VCS and svn displays some informations (keywords) from SVN.



              usepackage{ellipsis}


              corrects dots



              DeclarePairedDelimiter{abs}{lvert}{rvert}
              DeclarePairedDelimiter{norm}{lVert}{rVert}


              These are the definitions for absolute value and norm.



              SVN $LastChangedRevision$
              SVN $LastChangedDate$





              share|improve this answer



















              • 26




                "one thing (in this case, package) per answer"
                – Jukka Suomela
                Jul 29 '10 at 19:02






              • 4




                Could you break this up into multiple answers please, so they can be voted on? Having a dozen answers is ok!
                – ShreevatsaR
                Jul 30 '10 at 14:41










              • Excellent list with helpful commentary. I will add the one for old and deprecated things to my list. Thanks!
                – DJP
                Jul 30 '11 at 20:31






              • 3




                It is usually recommended to load hyperref last.
                – Alex Hirzel
                May 1 '12 at 20:20















              1 2
              3
              next



              Your Answer








              StackExchange.ready(function() {
              var channelOptions = {
              tags: "".split(" "),
              id: "85"
              };
              initTagRenderer("".split(" "), "".split(" "), channelOptions);

              StackExchange.using("externalEditor", function() {
              // Have to fire editor after snippets, if snippets enabled
              if (StackExchange.settings.snippets.snippetsEnabled) {
              StackExchange.using("snippets", function() {
              createEditor();
              });
              }
              else {
              createEditor();
              }
              });

              function createEditor() {
              StackExchange.prepareEditor({
              heartbeatType: 'answer',
              autoActivateHeartbeat: false,
              convertImagesToLinks: false,
              noModals: true,
              showLowRepImageUploadWarning: true,
              reputationToPostImages: null,
              bindNavPrevention: true,
              postfix: "",
              imageUploader: {
              brandingHtml: "Powered by u003ca class="icon-imgur-white" href="https://imgur.com/"u003eu003c/au003e",
              contentPolicyHtml: "User contributions licensed under u003ca href="https://creativecommons.org/licenses/by-sa/3.0/"u003ecc by-sa 3.0 with attribution requiredu003c/au003e u003ca href="https://stackoverflow.com/legal/content-policy"u003e(content policy)u003c/au003e",
              allowUrls: true
              },
              onDemand: false,
              discardSelector: ".discard-answer"
              ,immediatelyShowMarkdownHelp:true
              });


              }
              });














              draft saved

              draft discarded


















              StackExchange.ready(
              function () {
              StackExchange.openid.initPostLogin('.new-post-login', 'https%3a%2f%2ftex.stackexchange.com%2fquestions%2f553%2fwhat-packages-do-people-load-by-default-in-latex%23new-answer', 'question_page');
              }
              );

              Post as a guest















              Required, but never shown




















              StackExchange.ready(function () {
              $("#show-editor-button input, #show-editor-button button").click(function () {
              var showEditor = function() {
              $("#show-editor-button").hide();
              $("#post-form").removeClass("dno");
              StackExchange.editor.finallyInit();
              };

              var useFancy = $(this).data('confirm-use-fancy');
              if(useFancy == 'True') {
              var popupTitle = $(this).data('confirm-fancy-title');
              var popupBody = $(this).data('confirm-fancy-body');
              var popupAccept = $(this).data('confirm-fancy-accept-button');

              $(this).loadPopup({
              url: '/post/self-answer-popup',
              loaded: function(popup) {
              var pTitle = $(popup).find('h2');
              var pBody = $(popup).find('.popup-body');
              var pSubmit = $(popup).find('.popup-submit');

              pTitle.text(popupTitle);
              pBody.html(popupBody);
              pSubmit.val(popupAccept).click(showEditor);
              }
              })
              } else{
              var confirmText = $(this).data('confirm-text');
              if (confirmText ? confirm(confirmText) : true) {
              showEditor();
              }
              }
              });
              });






              63 Answers
              63






              active

              oldest

              votes








              63 Answers
              63






              active

              oldest

              votes









              active

              oldest

              votes






              active

              oldest

              votes








              1 2
              3
              next










              246














              I almost always load microtype. It plays with ever-so-slightly shrinking and stretching of the fonts and with the extent to which text protrudes into the margins in a way that yields results that look better, that have fewer instances of hyphenation, and fewer overfull hboxes. It doesn't work with latex, you have to use pdflatex instead. It also works with lualatex and (protrusion only) with xelatex.






              share|improve this answer



















              • 14




                You may want to use usepackage[stretch=10]{microtype}, which allows font expansion up to 1% (default is 2%).
                – lockstep
                Aug 6 '10 at 12:03






              • 69




                Can we have an example of with versus without?
                – levesque
                Nov 15 '10 at 18:28






              • 9




                there's a nice example in the documentation for microtype mirror.ctan.org/macros/latex/contrib/microtype/microtype.pdf, though it requires adobe acrobat for the inline examples
                – Noah
                Aug 12 '11 at 22:37






              • 11




                Here is another example.
                – Juri Robl
                Oct 11 '12 at 11:13






              • 4




                Here is a superb page of examples with and without microtype: khirevich.com/latex/microtype
                – Travis Bemrose
                Aug 19 '16 at 4:03
















              246














              I almost always load microtype. It plays with ever-so-slightly shrinking and stretching of the fonts and with the extent to which text protrudes into the margins in a way that yields results that look better, that have fewer instances of hyphenation, and fewer overfull hboxes. It doesn't work with latex, you have to use pdflatex instead. It also works with lualatex and (protrusion only) with xelatex.






              share|improve this answer



















              • 14




                You may want to use usepackage[stretch=10]{microtype}, which allows font expansion up to 1% (default is 2%).
                – lockstep
                Aug 6 '10 at 12:03






              • 69




                Can we have an example of with versus without?
                – levesque
                Nov 15 '10 at 18:28






              • 9




                there's a nice example in the documentation for microtype mirror.ctan.org/macros/latex/contrib/microtype/microtype.pdf, though it requires adobe acrobat for the inline examples
                – Noah
                Aug 12 '11 at 22:37






              • 11




                Here is another example.
                – Juri Robl
                Oct 11 '12 at 11:13






              • 4




                Here is a superb page of examples with and without microtype: khirevich.com/latex/microtype
                – Travis Bemrose
                Aug 19 '16 at 4:03














              246












              246








              246






              I almost always load microtype. It plays with ever-so-slightly shrinking and stretching of the fonts and with the extent to which text protrudes into the margins in a way that yields results that look better, that have fewer instances of hyphenation, and fewer overfull hboxes. It doesn't work with latex, you have to use pdflatex instead. It also works with lualatex and (protrusion only) with xelatex.






              share|improve this answer














              I almost always load microtype. It plays with ever-so-slightly shrinking and stretching of the fonts and with the extent to which text protrudes into the margins in a way that yields results that look better, that have fewer instances of hyphenation, and fewer overfull hboxes. It doesn't work with latex, you have to use pdflatex instead. It also works with lualatex and (protrusion only) with xelatex.







              share|improve this answer














              share|improve this answer



              share|improve this answer








              edited Feb 26 '14 at 3:29


























              community wiki





              3 revs, 3 users 50%
              vanden









              • 14




                You may want to use usepackage[stretch=10]{microtype}, which allows font expansion up to 1% (default is 2%).
                – lockstep
                Aug 6 '10 at 12:03






              • 69




                Can we have an example of with versus without?
                – levesque
                Nov 15 '10 at 18:28






              • 9




                there's a nice example in the documentation for microtype mirror.ctan.org/macros/latex/contrib/microtype/microtype.pdf, though it requires adobe acrobat for the inline examples
                – Noah
                Aug 12 '11 at 22:37






              • 11




                Here is another example.
                – Juri Robl
                Oct 11 '12 at 11:13






              • 4




                Here is a superb page of examples with and without microtype: khirevich.com/latex/microtype
                – Travis Bemrose
                Aug 19 '16 at 4:03














              • 14




                You may want to use usepackage[stretch=10]{microtype}, which allows font expansion up to 1% (default is 2%).
                – lockstep
                Aug 6 '10 at 12:03






              • 69




                Can we have an example of with versus without?
                – levesque
                Nov 15 '10 at 18:28






              • 9




                there's a nice example in the documentation for microtype mirror.ctan.org/macros/latex/contrib/microtype/microtype.pdf, though it requires adobe acrobat for the inline examples
                – Noah
                Aug 12 '11 at 22:37






              • 11




                Here is another example.
                – Juri Robl
                Oct 11 '12 at 11:13






              • 4




                Here is a superb page of examples with and without microtype: khirevich.com/latex/microtype
                – Travis Bemrose
                Aug 19 '16 at 4:03








              14




              14




              You may want to use usepackage[stretch=10]{microtype}, which allows font expansion up to 1% (default is 2%).
              – lockstep
              Aug 6 '10 at 12:03




              You may want to use usepackage[stretch=10]{microtype}, which allows font expansion up to 1% (default is 2%).
              – lockstep
              Aug 6 '10 at 12:03




              69




              69




              Can we have an example of with versus without?
              – levesque
              Nov 15 '10 at 18:28




              Can we have an example of with versus without?
              – levesque
              Nov 15 '10 at 18:28




              9




              9




              there's a nice example in the documentation for microtype mirror.ctan.org/macros/latex/contrib/microtype/microtype.pdf, though it requires adobe acrobat for the inline examples
              – Noah
              Aug 12 '11 at 22:37




              there's a nice example in the documentation for microtype mirror.ctan.org/macros/latex/contrib/microtype/microtype.pdf, though it requires adobe acrobat for the inline examples
              – Noah
              Aug 12 '11 at 22:37




              11




              11




              Here is another example.
              – Juri Robl
              Oct 11 '12 at 11:13




              Here is another example.
              – Juri Robl
              Oct 11 '12 at 11:13




              4




              4




              Here is a superb page of examples with and without microtype: khirevich.com/latex/microtype
              – Travis Bemrose
              Aug 19 '16 at 4:03




              Here is a superb page of examples with and without microtype: khirevich.com/latex/microtype
              – Travis Bemrose
              Aug 19 '16 at 4:03











              199














              The family of AMS math packages. At least amsmath and amssymb. Also amsthm if I need theorems and the class I'm using doesn't already define them.



              Particularly for writing equations, the AMS packages define a rich set of environments to group and align formulas in many different and useful ways. I also like that it encourages the use of semantic commands (e.g. the cases environment) over syntactic commands (e.g. a left{ followed by an array).



              Its documentation can be found running texdoc amsldoc on a command line.






              share|improve this answer



















              • 5




                In particular, amsthm provides an easy way to set up different theorem styles, amsmath provides the text command, and amssymb contains several often-used symbols.
                – András Salamon
                Jul 29 '10 at 12:40






              • 22




                +1 for the (oblique) reference to texdoc. I only discovered that recently and I wonder how I ever lived without it!
                – Loop Space
                Jul 29 '10 at 18:08






              • 6




                I believe amssymb loads amsfonts. There's rarely any need to load it yourself.
                – TH.
                Sep 11 '10 at 9:13






              • 5




                Note that the ams math packages are loaded automatically if you use one of their document classes, such as amsart.
                – Erik P.
                Jan 18 '12 at 19:08






              • 12




                Instead of loading amsmath I usually load mathtools. It is based on amsmath and loads it automatically. Moreover it fixes some deficiencies of the amsmath package and provides additional useful commands such as coloneqq.
                – Stan
                Aug 24 '14 at 9:53


















              199














              The family of AMS math packages. At least amsmath and amssymb. Also amsthm if I need theorems and the class I'm using doesn't already define them.



              Particularly for writing equations, the AMS packages define a rich set of environments to group and align formulas in many different and useful ways. I also like that it encourages the use of semantic commands (e.g. the cases environment) over syntactic commands (e.g. a left{ followed by an array).



              Its documentation can be found running texdoc amsldoc on a command line.






              share|improve this answer



















              • 5




                In particular, amsthm provides an easy way to set up different theorem styles, amsmath provides the text command, and amssymb contains several often-used symbols.
                – András Salamon
                Jul 29 '10 at 12:40






              • 22




                +1 for the (oblique) reference to texdoc. I only discovered that recently and I wonder how I ever lived without it!
                – Loop Space
                Jul 29 '10 at 18:08






              • 6




                I believe amssymb loads amsfonts. There's rarely any need to load it yourself.
                – TH.
                Sep 11 '10 at 9:13






              • 5




                Note that the ams math packages are loaded automatically if you use one of their document classes, such as amsart.
                – Erik P.
                Jan 18 '12 at 19:08






              • 12




                Instead of loading amsmath I usually load mathtools. It is based on amsmath and loads it automatically. Moreover it fixes some deficiencies of the amsmath package and provides additional useful commands such as coloneqq.
                – Stan
                Aug 24 '14 at 9:53
















              199












              199








              199






              The family of AMS math packages. At least amsmath and amssymb. Also amsthm if I need theorems and the class I'm using doesn't already define them.



              Particularly for writing equations, the AMS packages define a rich set of environments to group and align formulas in many different and useful ways. I also like that it encourages the use of semantic commands (e.g. the cases environment) over syntactic commands (e.g. a left{ followed by an array).



              Its documentation can be found running texdoc amsldoc on a command line.






              share|improve this answer














              The family of AMS math packages. At least amsmath and amssymb. Also amsthm if I need theorems and the class I'm using doesn't already define them.



              Particularly for writing equations, the AMS packages define a rich set of environments to group and align formulas in many different and useful ways. I also like that it encourages the use of semantic commands (e.g. the cases environment) over syntactic commands (e.g. a left{ followed by an array).



              Its documentation can be found running texdoc amsldoc on a command line.







              share|improve this answer














              share|improve this answer



              share|improve this answer








              edited Nov 15 '10 at 18:05


























              community wiki





              2 revs, 2 users 67%
              Juan A. Navarro










              • 5




                In particular, amsthm provides an easy way to set up different theorem styles, amsmath provides the text command, and amssymb contains several often-used symbols.
                – András Salamon
                Jul 29 '10 at 12:40






              • 22




                +1 for the (oblique) reference to texdoc. I only discovered that recently and I wonder how I ever lived without it!
                – Loop Space
                Jul 29 '10 at 18:08






              • 6




                I believe amssymb loads amsfonts. There's rarely any need to load it yourself.
                – TH.
                Sep 11 '10 at 9:13






              • 5




                Note that the ams math packages are loaded automatically if you use one of their document classes, such as amsart.
                – Erik P.
                Jan 18 '12 at 19:08






              • 12




                Instead of loading amsmath I usually load mathtools. It is based on amsmath and loads it automatically. Moreover it fixes some deficiencies of the amsmath package and provides additional useful commands such as coloneqq.
                – Stan
                Aug 24 '14 at 9:53
















              • 5




                In particular, amsthm provides an easy way to set up different theorem styles, amsmath provides the text command, and amssymb contains several often-used symbols.
                – András Salamon
                Jul 29 '10 at 12:40






              • 22




                +1 for the (oblique) reference to texdoc. I only discovered that recently and I wonder how I ever lived without it!
                – Loop Space
                Jul 29 '10 at 18:08






              • 6




                I believe amssymb loads amsfonts. There's rarely any need to load it yourself.
                – TH.
                Sep 11 '10 at 9:13






              • 5




                Note that the ams math packages are loaded automatically if you use one of their document classes, such as amsart.
                – Erik P.
                Jan 18 '12 at 19:08






              • 12




                Instead of loading amsmath I usually load mathtools. It is based on amsmath and loads it automatically. Moreover it fixes some deficiencies of the amsmath package and provides additional useful commands such as coloneqq.
                – Stan
                Aug 24 '14 at 9:53










              5




              5




              In particular, amsthm provides an easy way to set up different theorem styles, amsmath provides the text command, and amssymb contains several often-used symbols.
              – András Salamon
              Jul 29 '10 at 12:40




              In particular, amsthm provides an easy way to set up different theorem styles, amsmath provides the text command, and amssymb contains several often-used symbols.
              – András Salamon
              Jul 29 '10 at 12:40




              22




              22




              +1 for the (oblique) reference to texdoc. I only discovered that recently and I wonder how I ever lived without it!
              – Loop Space
              Jul 29 '10 at 18:08




              +1 for the (oblique) reference to texdoc. I only discovered that recently and I wonder how I ever lived without it!
              – Loop Space
              Jul 29 '10 at 18:08




              6




              6




              I believe amssymb loads amsfonts. There's rarely any need to load it yourself.
              – TH.
              Sep 11 '10 at 9:13




              I believe amssymb loads amsfonts. There's rarely any need to load it yourself.
              – TH.
              Sep 11 '10 at 9:13




              5




              5




              Note that the ams math packages are loaded automatically if you use one of their document classes, such as amsart.
              – Erik P.
              Jan 18 '12 at 19:08




              Note that the ams math packages are loaded automatically if you use one of their document classes, such as amsart.
              – Erik P.
              Jan 18 '12 at 19:08




              12




              12




              Instead of loading amsmath I usually load mathtools. It is based on amsmath and loads it automatically. Moreover it fixes some deficiencies of the amsmath package and provides additional useful commands such as coloneqq.
              – Stan
              Aug 24 '14 at 9:53






              Instead of loading amsmath I usually load mathtools. It is based on amsmath and loads it automatically. Moreover it fixes some deficiencies of the amsmath package and provides additional useful commands such as coloneqq.
              – Stan
              Aug 24 '14 at 9:53













              171














              I use hyperref for setting PDF metadata and to create links, both within the document and for clickable URLs. Even Elsevier has used urlbst to update their bibliography style to support URLs and DOIs; hyperref does the actual work of rendering url = and doi = BibTeX fields into clickable PDF links.






              share|improve this answer























              • related question: tex.stackexchange.com/questions/1863/…
                – matth
                Jun 26 '12 at 21:25
















              171














              I use hyperref for setting PDF metadata and to create links, both within the document and for clickable URLs. Even Elsevier has used urlbst to update their bibliography style to support URLs and DOIs; hyperref does the actual work of rendering url = and doi = BibTeX fields into clickable PDF links.






              share|improve this answer























              • related question: tex.stackexchange.com/questions/1863/…
                – matth
                Jun 26 '12 at 21:25














              171












              171








              171






              I use hyperref for setting PDF metadata and to create links, both within the document and for clickable URLs. Even Elsevier has used urlbst to update their bibliography style to support URLs and DOIs; hyperref does the actual work of rendering url = and doi = BibTeX fields into clickable PDF links.






              share|improve this answer














              I use hyperref for setting PDF metadata and to create links, both within the document and for clickable URLs. Even Elsevier has used urlbst to update their bibliography style to support URLs and DOIs; hyperref does the actual work of rendering url = and doi = BibTeX fields into clickable PDF links.







              share|improve this answer














              share|improve this answer



              share|improve this answer








              answered Jul 29 '10 at 12:52


























              community wiki





              András Salamon













              • related question: tex.stackexchange.com/questions/1863/…
                – matth
                Jun 26 '12 at 21:25


















              • related question: tex.stackexchange.com/questions/1863/…
                – matth
                Jun 26 '12 at 21:25
















              related question: tex.stackexchange.com/questions/1863/…
              – matth
              Jun 26 '12 at 21:25




              related question: tex.stackexchange.com/questions/1863/…
              – matth
              Jun 26 '12 at 21:25











              155














              For citations and bibliographies, biblatex is the package of my choice. Key points:




              • biblatex includes a wide variety of built-in citation/bibliography styles (numeric, alphabetic, author-year, author-title, verbose [full in-text-citations], with numerous variants for each one). A number of custom styles have been published.


              • Modifications of the built-in or custom styles can be accomplished using LaTeX macros instead of having to resort to the BibTeX programming language.


              • biblatex offers well-nigh every feature of other bibliography-related LaTeX packages (e.g. multiple/subdivided bibliographies, sorted/compressed citations, entry sets, ibidem functionality, back references). If a feature is not included, chances are high it is on the package authors' to-do list.


              • The babel package is supported, and biblatex comes with localization files for about a dozen languages (with the list still growing).


              • Although the current version of biblatex (2.8a) still allows to use BibTeX as a database backend, by default it cooperates with Biber which supports bibliographies using Unicode. Biber (currently at version 1.8) is included in TeX Live and MiKTeX. Many features introduced since biblatex 1.1 (e.g., advanced name disambiguation, smart crossref data inheritance, configurable sorting schemes, dynamic datasource modification) are "Biber only".







              share|improve this answer



















              • 7




                Nevertheless one should append about the usage of biblatex that some papers do not accept its usage. See: Biblatex: submitting to a journal
                – strpeter
                Jan 16 '14 at 9:25


















              155














              For citations and bibliographies, biblatex is the package of my choice. Key points:




              • biblatex includes a wide variety of built-in citation/bibliography styles (numeric, alphabetic, author-year, author-title, verbose [full in-text-citations], with numerous variants for each one). A number of custom styles have been published.


              • Modifications of the built-in or custom styles can be accomplished using LaTeX macros instead of having to resort to the BibTeX programming language.


              • biblatex offers well-nigh every feature of other bibliography-related LaTeX packages (e.g. multiple/subdivided bibliographies, sorted/compressed citations, entry sets, ibidem functionality, back references). If a feature is not included, chances are high it is on the package authors' to-do list.


              • The babel package is supported, and biblatex comes with localization files for about a dozen languages (with the list still growing).


              • Although the current version of biblatex (2.8a) still allows to use BibTeX as a database backend, by default it cooperates with Biber which supports bibliographies using Unicode. Biber (currently at version 1.8) is included in TeX Live and MiKTeX. Many features introduced since biblatex 1.1 (e.g., advanced name disambiguation, smart crossref data inheritance, configurable sorting schemes, dynamic datasource modification) are "Biber only".







              share|improve this answer



















              • 7




                Nevertheless one should append about the usage of biblatex that some papers do not accept its usage. See: Biblatex: submitting to a journal
                – strpeter
                Jan 16 '14 at 9:25
















              155












              155








              155






              For citations and bibliographies, biblatex is the package of my choice. Key points:




              • biblatex includes a wide variety of built-in citation/bibliography styles (numeric, alphabetic, author-year, author-title, verbose [full in-text-citations], with numerous variants for each one). A number of custom styles have been published.


              • Modifications of the built-in or custom styles can be accomplished using LaTeX macros instead of having to resort to the BibTeX programming language.


              • biblatex offers well-nigh every feature of other bibliography-related LaTeX packages (e.g. multiple/subdivided bibliographies, sorted/compressed citations, entry sets, ibidem functionality, back references). If a feature is not included, chances are high it is on the package authors' to-do list.


              • The babel package is supported, and biblatex comes with localization files for about a dozen languages (with the list still growing).


              • Although the current version of biblatex (2.8a) still allows to use BibTeX as a database backend, by default it cooperates with Biber which supports bibliographies using Unicode. Biber (currently at version 1.8) is included in TeX Live and MiKTeX. Many features introduced since biblatex 1.1 (e.g., advanced name disambiguation, smart crossref data inheritance, configurable sorting schemes, dynamic datasource modification) are "Biber only".







              share|improve this answer














              For citations and bibliographies, biblatex is the package of my choice. Key points:




              • biblatex includes a wide variety of built-in citation/bibliography styles (numeric, alphabetic, author-year, author-title, verbose [full in-text-citations], with numerous variants for each one). A number of custom styles have been published.


              • Modifications of the built-in or custom styles can be accomplished using LaTeX macros instead of having to resort to the BibTeX programming language.


              • biblatex offers well-nigh every feature of other bibliography-related LaTeX packages (e.g. multiple/subdivided bibliographies, sorted/compressed citations, entry sets, ibidem functionality, back references). If a feature is not included, chances are high it is on the package authors' to-do list.


              • The babel package is supported, and biblatex comes with localization files for about a dozen languages (with the list still growing).


              • Although the current version of biblatex (2.8a) still allows to use BibTeX as a database backend, by default it cooperates with Biber which supports bibliographies using Unicode. Biber (currently at version 1.8) is included in TeX Live and MiKTeX. Many features introduced since biblatex 1.1 (e.g., advanced name disambiguation, smart crossref data inheritance, configurable sorting schemes, dynamic datasource modification) are "Biber only".








              share|improve this answer














              share|improve this answer



              share|improve this answer








              edited Mar 16 '14 at 16:44


























              community wiki





              16 revs, 2 users 88%
              lockstep









              • 7




                Nevertheless one should append about the usage of biblatex that some papers do not accept its usage. See: Biblatex: submitting to a journal
                – strpeter
                Jan 16 '14 at 9:25
















              • 7




                Nevertheless one should append about the usage of biblatex that some papers do not accept its usage. See: Biblatex: submitting to a journal
                – strpeter
                Jan 16 '14 at 9:25










              7




              7




              Nevertheless one should append about the usage of biblatex that some papers do not accept its usage. See: Biblatex: submitting to a journal
              – strpeter
              Jan 16 '14 at 9:25






              Nevertheless one should append about the usage of biblatex that some papers do not accept its usage. See: Biblatex: submitting to a journal
              – strpeter
              Jan 16 '14 at 9:25













              138














              The todonotes package is a must have in all my documents.



              usepackage{todonotes}


              The package enables you to insert small notes in the text marking things to do in the document. Something like



              todo{Rewrite this answer ldots}


              At any location in the document a list of the inserted notes can be generated with the



              listoftodos 


              command.






              share|improve this answer



















              • 11




                For multiuser comment support, and configurability with regard to the kinds of notes/themes available, the fixme package is quite nice (I use it quite regularly).
                – Mark
                Mar 25 '11 at 22:29










              • todonotes also supports colors and missing graphics.
                – ℝaphink
                Jun 16 '11 at 10:42










              • I find todonote invaluable as I prepare syllabai and course material for the upcoming term. Because I cannot do everything at one sitting I put a todo note whenever i find something I have to wait to do. I also use it during the semester to highlight to the students anything which was changed after the documents were first published.
                – R. Schumacher
                May 1 '12 at 20:51








              • 4




                Personally I use an editor which automatically highlights and groups in the "structure" window any comment that begins with %TODO: Works better for me because you don't have anything in your compiled document giving away the fact that it still has TODOs around.
                – Dom
                Jun 12 '13 at 10:53






              • 12




                Has anyone done a comparison between easy-todo, fixme, fixmetodonotes, todo, and todonotes?
                – Ari Brodsky
                Nov 20 '13 at 1:12
















              138














              The todonotes package is a must have in all my documents.



              usepackage{todonotes}


              The package enables you to insert small notes in the text marking things to do in the document. Something like



              todo{Rewrite this answer ldots}


              At any location in the document a list of the inserted notes can be generated with the



              listoftodos 


              command.






              share|improve this answer



















              • 11




                For multiuser comment support, and configurability with regard to the kinds of notes/themes available, the fixme package is quite nice (I use it quite regularly).
                – Mark
                Mar 25 '11 at 22:29










              • todonotes also supports colors and missing graphics.
                – ℝaphink
                Jun 16 '11 at 10:42










              • I find todonote invaluable as I prepare syllabai and course material for the upcoming term. Because I cannot do everything at one sitting I put a todo note whenever i find something I have to wait to do. I also use it during the semester to highlight to the students anything which was changed after the documents were first published.
                – R. Schumacher
                May 1 '12 at 20:51








              • 4




                Personally I use an editor which automatically highlights and groups in the "structure" window any comment that begins with %TODO: Works better for me because you don't have anything in your compiled document giving away the fact that it still has TODOs around.
                – Dom
                Jun 12 '13 at 10:53






              • 12




                Has anyone done a comparison between easy-todo, fixme, fixmetodonotes, todo, and todonotes?
                – Ari Brodsky
                Nov 20 '13 at 1:12














              138












              138








              138






              The todonotes package is a must have in all my documents.



              usepackage{todonotes}


              The package enables you to insert small notes in the text marking things to do in the document. Something like



              todo{Rewrite this answer ldots}


              At any location in the document a list of the inserted notes can be generated with the



              listoftodos 


              command.






              share|improve this answer














              The todonotes package is a must have in all my documents.



              usepackage{todonotes}


              The package enables you to insert small notes in the text marking things to do in the document. Something like



              todo{Rewrite this answer ldots}


              At any location in the document a list of the inserted notes can be generated with the



              listoftodos 


              command.







              share|improve this answer














              share|improve this answer



              share|improve this answer








              answered Oct 10 '10 at 8:19


























              community wiki





              midtiby









              • 11




                For multiuser comment support, and configurability with regard to the kinds of notes/themes available, the fixme package is quite nice (I use it quite regularly).
                – Mark
                Mar 25 '11 at 22:29










              • todonotes also supports colors and missing graphics.
                – ℝaphink
                Jun 16 '11 at 10:42










              • I find todonote invaluable as I prepare syllabai and course material for the upcoming term. Because I cannot do everything at one sitting I put a todo note whenever i find something I have to wait to do. I also use it during the semester to highlight to the students anything which was changed after the documents were first published.
                – R. Schumacher
                May 1 '12 at 20:51








              • 4




                Personally I use an editor which automatically highlights and groups in the "structure" window any comment that begins with %TODO: Works better for me because you don't have anything in your compiled document giving away the fact that it still has TODOs around.
                – Dom
                Jun 12 '13 at 10:53






              • 12




                Has anyone done a comparison between easy-todo, fixme, fixmetodonotes, todo, and todonotes?
                – Ari Brodsky
                Nov 20 '13 at 1:12














              • 11




                For multiuser comment support, and configurability with regard to the kinds of notes/themes available, the fixme package is quite nice (I use it quite regularly).
                – Mark
                Mar 25 '11 at 22:29










              • todonotes also supports colors and missing graphics.
                – ℝaphink
                Jun 16 '11 at 10:42










              • I find todonote invaluable as I prepare syllabai and course material for the upcoming term. Because I cannot do everything at one sitting I put a todo note whenever i find something I have to wait to do. I also use it during the semester to highlight to the students anything which was changed after the documents were first published.
                – R. Schumacher
                May 1 '12 at 20:51








              • 4




                Personally I use an editor which automatically highlights and groups in the "structure" window any comment that begins with %TODO: Works better for me because you don't have anything in your compiled document giving away the fact that it still has TODOs around.
                – Dom
                Jun 12 '13 at 10:53






              • 12




                Has anyone done a comparison between easy-todo, fixme, fixmetodonotes, todo, and todonotes?
                – Ari Brodsky
                Nov 20 '13 at 1:12








              11




              11




              For multiuser comment support, and configurability with regard to the kinds of notes/themes available, the fixme package is quite nice (I use it quite regularly).
              – Mark
              Mar 25 '11 at 22:29




              For multiuser comment support, and configurability with regard to the kinds of notes/themes available, the fixme package is quite nice (I use it quite regularly).
              – Mark
              Mar 25 '11 at 22:29












              todonotes also supports colors and missing graphics.
              – ℝaphink
              Jun 16 '11 at 10:42




              todonotes also supports colors and missing graphics.
              – ℝaphink
              Jun 16 '11 at 10:42












              I find todonote invaluable as I prepare syllabai and course material for the upcoming term. Because I cannot do everything at one sitting I put a todo note whenever i find something I have to wait to do. I also use it during the semester to highlight to the students anything which was changed after the documents were first published.
              – R. Schumacher
              May 1 '12 at 20:51






              I find todonote invaluable as I prepare syllabai and course material for the upcoming term. Because I cannot do everything at one sitting I put a todo note whenever i find something I have to wait to do. I also use it during the semester to highlight to the students anything which was changed after the documents were first published.
              – R. Schumacher
              May 1 '12 at 20:51






              4




              4




              Personally I use an editor which automatically highlights and groups in the "structure" window any comment that begins with %TODO: Works better for me because you don't have anything in your compiled document giving away the fact that it still has TODOs around.
              – Dom
              Jun 12 '13 at 10:53




              Personally I use an editor which automatically highlights and groups in the "structure" window any comment that begins with %TODO: Works better for me because you don't have anything in your compiled document giving away the fact that it still has TODOs around.
              – Dom
              Jun 12 '13 at 10:53




              12




              12




              Has anyone done a comparison between easy-todo, fixme, fixmetodonotes, todo, and todonotes?
              – Ari Brodsky
              Nov 20 '13 at 1:12




              Has anyone done a comparison between easy-todo, fixme, fixmetodonotes, todo, and todonotes?
              – Ari Brodsky
              Nov 20 '13 at 1:12











              132














              One package that’s really general purpose is nag: It doesn’t do anything, per se, it just warns when you accidentally use deprecated LaTeX constructs from l2tabu (English / French / German / Italian / Spanish documentation).



              From the documentation:




              Old habits die hard. All the same, there are commands, classes and packages which are outdated and superseded. nag provides routines to warn the user about the use of those. As an example, we provide an extension that detects many of the “sins” described in l2tabu.




              Therefore, I now always have the following in my header (before the documentclass, thanks qbi):



              RequirePackage[l2tabu, orthodox]{nag}


              It’s a bit like having use strict; in Perl: a useful best practice.






              share|improve this answer



















              • 22




                Somewhat better is RequirePackage[l2tabu,orthodox]{nag} before documentclass. The package docu also recommends this.
                – qbi
                Jul 29 '10 at 18:40






              • 5




                This package sounds useful. However, when I tested it with a large project, I started to get the message "Label(s) may have changed. Rerun to get cross-references right." no matter how many times I re-run Latex.
                – Jukka Suomela
                Jul 31 '10 at 9:36
















              132














              One package that’s really general purpose is nag: It doesn’t do anything, per se, it just warns when you accidentally use deprecated LaTeX constructs from l2tabu (English / French / German / Italian / Spanish documentation).



              From the documentation:




              Old habits die hard. All the same, there are commands, classes and packages which are outdated and superseded. nag provides routines to warn the user about the use of those. As an example, we provide an extension that detects many of the “sins” described in l2tabu.




              Therefore, I now always have the following in my header (before the documentclass, thanks qbi):



              RequirePackage[l2tabu, orthodox]{nag}


              It’s a bit like having use strict; in Perl: a useful best practice.






              share|improve this answer



















              • 22




                Somewhat better is RequirePackage[l2tabu,orthodox]{nag} before documentclass. The package docu also recommends this.
                – qbi
                Jul 29 '10 at 18:40






              • 5




                This package sounds useful. However, when I tested it with a large project, I started to get the message "Label(s) may have changed. Rerun to get cross-references right." no matter how many times I re-run Latex.
                – Jukka Suomela
                Jul 31 '10 at 9:36














              132












              132








              132






              One package that’s really general purpose is nag: It doesn’t do anything, per se, it just warns when you accidentally use deprecated LaTeX constructs from l2tabu (English / French / German / Italian / Spanish documentation).



              From the documentation:




              Old habits die hard. All the same, there are commands, classes and packages which are outdated and superseded. nag provides routines to warn the user about the use of those. As an example, we provide an extension that detects many of the “sins” described in l2tabu.




              Therefore, I now always have the following in my header (before the documentclass, thanks qbi):



              RequirePackage[l2tabu, orthodox]{nag}


              It’s a bit like having use strict; in Perl: a useful best practice.






              share|improve this answer














              One package that’s really general purpose is nag: It doesn’t do anything, per se, it just warns when you accidentally use deprecated LaTeX constructs from l2tabu (English / French / German / Italian / Spanish documentation).



              From the documentation:




              Old habits die hard. All the same, there are commands, classes and packages which are outdated and superseded. nag provides routines to warn the user about the use of those. As an example, we provide an extension that detects many of the “sins” described in l2tabu.




              Therefore, I now always have the following in my header (before the documentclass, thanks qbi):



              RequirePackage[l2tabu, orthodox]{nag}


              It’s a bit like having use strict; in Perl: a useful best practice.







              share|improve this answer














              share|improve this answer



              share|improve this answer








              edited Apr 9 '14 at 20:48


























              community wiki





              7 revs, 5 users 61%
              Konrad Rudolph









              • 22




                Somewhat better is RequirePackage[l2tabu,orthodox]{nag} before documentclass. The package docu also recommends this.
                – qbi
                Jul 29 '10 at 18:40






              • 5




                This package sounds useful. However, when I tested it with a large project, I started to get the message "Label(s) may have changed. Rerun to get cross-references right." no matter how many times I re-run Latex.
                – Jukka Suomela
                Jul 31 '10 at 9:36














              • 22




                Somewhat better is RequirePackage[l2tabu,orthodox]{nag} before documentclass. The package docu also recommends this.
                – qbi
                Jul 29 '10 at 18:40






              • 5




                This package sounds useful. However, when I tested it with a large project, I started to get the message "Label(s) may have changed. Rerun to get cross-references right." no matter how many times I re-run Latex.
                – Jukka Suomela
                Jul 31 '10 at 9:36








              22




              22




              Somewhat better is RequirePackage[l2tabu,orthodox]{nag} before documentclass. The package docu also recommends this.
              – qbi
              Jul 29 '10 at 18:40




              Somewhat better is RequirePackage[l2tabu,orthodox]{nag} before documentclass. The package docu also recommends this.
              – qbi
              Jul 29 '10 at 18:40




              5




              5




              This package sounds useful. However, when I tested it with a large project, I started to get the message "Label(s) may have changed. Rerun to get cross-references right." no matter how many times I re-run Latex.
              – Jukka Suomela
              Jul 31 '10 at 9:36




              This package sounds useful. However, when I tested it with a large project, I started to get the message "Label(s) may have changed. Rerun to get cross-references right." no matter how many times I re-run Latex.
              – Jukka Suomela
              Jul 31 '10 at 9:36











              114














              I'm surprised that no one has mentioned



              usepackage[margin=1in]{geometry} % set page margins automatically 


              This is in every document I write (with varying margins, of course.)






              share|improve this answer



















              • 13




                This is generally poor style. The design of the page is pretty involved and lots of thought has went into (La)TeX's default designs. If you're interested in just saving paper, consider the packages savetrees or fullpage.
                – Quadrescence
                Apr 16 '11 at 23:15








              • 23




                Both savetrees and fullpage change other things too; Anyway, the point of of the answer is that geometry is a must use package, no matter what margins you choose for it. The appropriateness of 1in margins also depends on the kind of documents you produce.
                – Alan Munn
                Apr 16 '11 at 23:38






              • 12




                It is not a must if you use a class from the KOMAscript bundle or memoir.
                – Sveinung
                Jan 13 '14 at 16:01










              • @Sveinung but it is a must if you have to comply with certain margins, because your supervisor/advisor/publisher orders you to do so.
                – Skillmon
                Dec 14 '18 at 20:08










              • @Skillmon No, because memoir and KOMA have their own ways to set margins, so geometry isn't needed, which is what Sveinung means, I suppose.
                – Alan Munn
                Dec 14 '18 at 20:10


















              114














              I'm surprised that no one has mentioned



              usepackage[margin=1in]{geometry} % set page margins automatically 


              This is in every document I write (with varying margins, of course.)






              share|improve this answer



















              • 13




                This is generally poor style. The design of the page is pretty involved and lots of thought has went into (La)TeX's default designs. If you're interested in just saving paper, consider the packages savetrees or fullpage.
                – Quadrescence
                Apr 16 '11 at 23:15








              • 23




                Both savetrees and fullpage change other things too; Anyway, the point of of the answer is that geometry is a must use package, no matter what margins you choose for it. The appropriateness of 1in margins also depends on the kind of documents you produce.
                – Alan Munn
                Apr 16 '11 at 23:38






              • 12




                It is not a must if you use a class from the KOMAscript bundle or memoir.
                – Sveinung
                Jan 13 '14 at 16:01










              • @Sveinung but it is a must if you have to comply with certain margins, because your supervisor/advisor/publisher orders you to do so.
                – Skillmon
                Dec 14 '18 at 20:08










              • @Skillmon No, because memoir and KOMA have their own ways to set margins, so geometry isn't needed, which is what Sveinung means, I suppose.
                – Alan Munn
                Dec 14 '18 at 20:10
















              114












              114








              114






              I'm surprised that no one has mentioned



              usepackage[margin=1in]{geometry} % set page margins automatically 


              This is in every document I write (with varying margins, of course.)






              share|improve this answer














              I'm surprised that no one has mentioned



              usepackage[margin=1in]{geometry} % set page margins automatically 


              This is in every document I write (with varying margins, of course.)







              share|improve this answer














              share|improve this answer



              share|improve this answer








              answered Dec 29 '10 at 0:08


























              community wiki





              Alan Munn









              • 13




                This is generally poor style. The design of the page is pretty involved and lots of thought has went into (La)TeX's default designs. If you're interested in just saving paper, consider the packages savetrees or fullpage.
                – Quadrescence
                Apr 16 '11 at 23:15








              • 23




                Both savetrees and fullpage change other things too; Anyway, the point of of the answer is that geometry is a must use package, no matter what margins you choose for it. The appropriateness of 1in margins also depends on the kind of documents you produce.
                – Alan Munn
                Apr 16 '11 at 23:38






              • 12




                It is not a must if you use a class from the KOMAscript bundle or memoir.
                – Sveinung
                Jan 13 '14 at 16:01










              • @Sveinung but it is a must if you have to comply with certain margins, because your supervisor/advisor/publisher orders you to do so.
                – Skillmon
                Dec 14 '18 at 20:08










              • @Skillmon No, because memoir and KOMA have their own ways to set margins, so geometry isn't needed, which is what Sveinung means, I suppose.
                – Alan Munn
                Dec 14 '18 at 20:10
















              • 13




                This is generally poor style. The design of the page is pretty involved and lots of thought has went into (La)TeX's default designs. If you're interested in just saving paper, consider the packages savetrees or fullpage.
                – Quadrescence
                Apr 16 '11 at 23:15








              • 23




                Both savetrees and fullpage change other things too; Anyway, the point of of the answer is that geometry is a must use package, no matter what margins you choose for it. The appropriateness of 1in margins also depends on the kind of documents you produce.
                – Alan Munn
                Apr 16 '11 at 23:38






              • 12




                It is not a must if you use a class from the KOMAscript bundle or memoir.
                – Sveinung
                Jan 13 '14 at 16:01










              • @Sveinung but it is a must if you have to comply with certain margins, because your supervisor/advisor/publisher orders you to do so.
                – Skillmon
                Dec 14 '18 at 20:08










              • @Skillmon No, because memoir and KOMA have their own ways to set margins, so geometry isn't needed, which is what Sveinung means, I suppose.
                – Alan Munn
                Dec 14 '18 at 20:10










              13




              13




              This is generally poor style. The design of the page is pretty involved and lots of thought has went into (La)TeX's default designs. If you're interested in just saving paper, consider the packages savetrees or fullpage.
              – Quadrescence
              Apr 16 '11 at 23:15






              This is generally poor style. The design of the page is pretty involved and lots of thought has went into (La)TeX's default designs. If you're interested in just saving paper, consider the packages savetrees or fullpage.
              – Quadrescence
              Apr 16 '11 at 23:15






              23




              23




              Both savetrees and fullpage change other things too; Anyway, the point of of the answer is that geometry is a must use package, no matter what margins you choose for it. The appropriateness of 1in margins also depends on the kind of documents you produce.
              – Alan Munn
              Apr 16 '11 at 23:38




              Both savetrees and fullpage change other things too; Anyway, the point of of the answer is that geometry is a must use package, no matter what margins you choose for it. The appropriateness of 1in margins also depends on the kind of documents you produce.
              – Alan Munn
              Apr 16 '11 at 23:38




              12




              12




              It is not a must if you use a class from the KOMAscript bundle or memoir.
              – Sveinung
              Jan 13 '14 at 16:01




              It is not a must if you use a class from the KOMAscript bundle or memoir.
              – Sveinung
              Jan 13 '14 at 16:01












              @Sveinung but it is a must if you have to comply with certain margins, because your supervisor/advisor/publisher orders you to do so.
              – Skillmon
              Dec 14 '18 at 20:08




              @Sveinung but it is a must if you have to comply with certain margins, because your supervisor/advisor/publisher orders you to do so.
              – Skillmon
              Dec 14 '18 at 20:08












              @Skillmon No, because memoir and KOMA have their own ways to set margins, so geometry isn't needed, which is what Sveinung means, I suppose.
              – Alan Munn
              Dec 14 '18 at 20:10






              @Skillmon No, because memoir and KOMA have their own ways to set margins, so geometry isn't needed, which is what Sveinung means, I suppose.
              – Alan Munn
              Dec 14 '18 at 20:10













              114














              Another essential package combination is



              usepackage{booktabs}
              usepackage{array}


              The booktabs package creates much nicer looking tables than the standard latex tables; the array package's ability to create custom columns is invaluable for formatting tabular material on a per-column basis.






              share|improve this answer



















              • 1




                I just discovered booktabs -- it is great!
                – Ben
                Jan 12 '11 at 22:37






              • 1




                @Ben Yes, it's a great package. If you visit my profile web link you can find my own list of essential packages.
                – Alan Munn
                Jan 12 '11 at 22:47
















              114














              Another essential package combination is



              usepackage{booktabs}
              usepackage{array}


              The booktabs package creates much nicer looking tables than the standard latex tables; the array package's ability to create custom columns is invaluable for formatting tabular material on a per-column basis.






              share|improve this answer



















              • 1




                I just discovered booktabs -- it is great!
                – Ben
                Jan 12 '11 at 22:37






              • 1




                @Ben Yes, it's a great package. If you visit my profile web link you can find my own list of essential packages.
                – Alan Munn
                Jan 12 '11 at 22:47














              114












              114








              114






              Another essential package combination is



              usepackage{booktabs}
              usepackage{array}


              The booktabs package creates much nicer looking tables than the standard latex tables; the array package's ability to create custom columns is invaluable for formatting tabular material on a per-column basis.






              share|improve this answer














              Another essential package combination is



              usepackage{booktabs}
              usepackage{array}


              The booktabs package creates much nicer looking tables than the standard latex tables; the array package's ability to create custom columns is invaluable for formatting tabular material on a per-column basis.







              share|improve this answer














              share|improve this answer



              share|improve this answer








              edited Jun 4 '12 at 16:19


























              community wiki





              2 revs, 2 users 91%
              Alan Munn










              • 1




                I just discovered booktabs -- it is great!
                – Ben
                Jan 12 '11 at 22:37






              • 1




                @Ben Yes, it's a great package. If you visit my profile web link you can find my own list of essential packages.
                – Alan Munn
                Jan 12 '11 at 22:47














              • 1




                I just discovered booktabs -- it is great!
                – Ben
                Jan 12 '11 at 22:37






              • 1




                @Ben Yes, it's a great package. If you visit my profile web link you can find my own list of essential packages.
                – Alan Munn
                Jan 12 '11 at 22:47








              1




              1




              I just discovered booktabs -- it is great!
              – Ben
              Jan 12 '11 at 22:37




              I just discovered booktabs -- it is great!
              – Ben
              Jan 12 '11 at 22:37




              1




              1




              @Ben Yes, it's a great package. If you visit my profile web link you can find my own list of essential packages.
              – Alan Munn
              Jan 12 '11 at 22:47




              @Ben Yes, it's a great package. If you visit my profile web link you can find my own list of essential packages.
              – Alan Munn
              Jan 12 '11 at 22:47











              113














              I nearly always use the tikz package. Once you learn how to draw with it, you can do almost any vector graphic you need.






              share|improve this answer























              • I have always used Inkscape for the production of my vector images, diagrams and whatsoever. Does tikz produce comparable diagrams? How much effort is involved?
                – levesque
                Nov 15 '10 at 18:28






              • 4




                You can produce almost any diagram with Tikz. Check the tikz examples page. texample.net/tikz/examples However, it is fairly complicated to get the hang on large diagrams since you have to type everything and nearly always you can't see what you are doing. But if you are using a Debian/KDE combination, you can use Ktikz/Qtikz which is really helpful since it compiles tikz code in real time.
                – fabikw
                Nov 16 '10 at 0:42








              • 35




                TikZ is awesome with a capital A. But load it by default? It takes up a lot of time and space. I would say only load it if you need it.
                – Matthew Leingang
                Nov 22 '10 at 12:53










              • It takes time, but nearly always I find I need to do something with it.
                – fabikw
                Nov 23 '10 at 1:11






              • 9




                @levesque: Tikz has a fairly steep learning curve, but it is beautifully documented and provides rich libraries. I find the vector graphics that I produce in tikz to be superior to those I produced in inkscape. It seems easier on my brain to stay in keyboard mode as well.
                – philosodad
                Dec 29 '10 at 4:56
















              113














              I nearly always use the tikz package. Once you learn how to draw with it, you can do almost any vector graphic you need.






              share|improve this answer























              • I have always used Inkscape for the production of my vector images, diagrams and whatsoever. Does tikz produce comparable diagrams? How much effort is involved?
                – levesque
                Nov 15 '10 at 18:28






              • 4




                You can produce almost any diagram with Tikz. Check the tikz examples page. texample.net/tikz/examples However, it is fairly complicated to get the hang on large diagrams since you have to type everything and nearly always you can't see what you are doing. But if you are using a Debian/KDE combination, you can use Ktikz/Qtikz which is really helpful since it compiles tikz code in real time.
                – fabikw
                Nov 16 '10 at 0:42








              • 35




                TikZ is awesome with a capital A. But load it by default? It takes up a lot of time and space. I would say only load it if you need it.
                – Matthew Leingang
                Nov 22 '10 at 12:53










              • It takes time, but nearly always I find I need to do something with it.
                – fabikw
                Nov 23 '10 at 1:11






              • 9




                @levesque: Tikz has a fairly steep learning curve, but it is beautifully documented and provides rich libraries. I find the vector graphics that I produce in tikz to be superior to those I produced in inkscape. It seems easier on my brain to stay in keyboard mode as well.
                – philosodad
                Dec 29 '10 at 4:56














              113












              113








              113






              I nearly always use the tikz package. Once you learn how to draw with it, you can do almost any vector graphic you need.






              share|improve this answer














              I nearly always use the tikz package. Once you learn how to draw with it, you can do almost any vector graphic you need.







              share|improve this answer














              share|improve this answer



              share|improve this answer








              answered Aug 5 '10 at 16:32


























              community wiki





              fabikw













              • I have always used Inkscape for the production of my vector images, diagrams and whatsoever. Does tikz produce comparable diagrams? How much effort is involved?
                – levesque
                Nov 15 '10 at 18:28






              • 4




                You can produce almost any diagram with Tikz. Check the tikz examples page. texample.net/tikz/examples However, it is fairly complicated to get the hang on large diagrams since you have to type everything and nearly always you can't see what you are doing. But if you are using a Debian/KDE combination, you can use Ktikz/Qtikz which is really helpful since it compiles tikz code in real time.
                – fabikw
                Nov 16 '10 at 0:42








              • 35




                TikZ is awesome with a capital A. But load it by default? It takes up a lot of time and space. I would say only load it if you need it.
                – Matthew Leingang
                Nov 22 '10 at 12:53










              • It takes time, but nearly always I find I need to do something with it.
                – fabikw
                Nov 23 '10 at 1:11






              • 9




                @levesque: Tikz has a fairly steep learning curve, but it is beautifully documented and provides rich libraries. I find the vector graphics that I produce in tikz to be superior to those I produced in inkscape. It seems easier on my brain to stay in keyboard mode as well.
                – philosodad
                Dec 29 '10 at 4:56


















              • I have always used Inkscape for the production of my vector images, diagrams and whatsoever. Does tikz produce comparable diagrams? How much effort is involved?
                – levesque
                Nov 15 '10 at 18:28






              • 4




                You can produce almost any diagram with Tikz. Check the tikz examples page. texample.net/tikz/examples However, it is fairly complicated to get the hang on large diagrams since you have to type everything and nearly always you can't see what you are doing. But if you are using a Debian/KDE combination, you can use Ktikz/Qtikz which is really helpful since it compiles tikz code in real time.
                – fabikw
                Nov 16 '10 at 0:42








              • 35




                TikZ is awesome with a capital A. But load it by default? It takes up a lot of time and space. I would say only load it if you need it.
                – Matthew Leingang
                Nov 22 '10 at 12:53










              • It takes time, but nearly always I find I need to do something with it.
                – fabikw
                Nov 23 '10 at 1:11






              • 9




                @levesque: Tikz has a fairly steep learning curve, but it is beautifully documented and provides rich libraries. I find the vector graphics that I produce in tikz to be superior to those I produced in inkscape. It seems easier on my brain to stay in keyboard mode as well.
                – philosodad
                Dec 29 '10 at 4:56
















              I have always used Inkscape for the production of my vector images, diagrams and whatsoever. Does tikz produce comparable diagrams? How much effort is involved?
              – levesque
              Nov 15 '10 at 18:28




              I have always used Inkscape for the production of my vector images, diagrams and whatsoever. Does tikz produce comparable diagrams? How much effort is involved?
              – levesque
              Nov 15 '10 at 18:28




              4




              4




              You can produce almost any diagram with Tikz. Check the tikz examples page. texample.net/tikz/examples However, it is fairly complicated to get the hang on large diagrams since you have to type everything and nearly always you can't see what you are doing. But if you are using a Debian/KDE combination, you can use Ktikz/Qtikz which is really helpful since it compiles tikz code in real time.
              – fabikw
              Nov 16 '10 at 0:42






              You can produce almost any diagram with Tikz. Check the tikz examples page. texample.net/tikz/examples However, it is fairly complicated to get the hang on large diagrams since you have to type everything and nearly always you can't see what you are doing. But if you are using a Debian/KDE combination, you can use Ktikz/Qtikz which is really helpful since it compiles tikz code in real time.
              – fabikw
              Nov 16 '10 at 0:42






              35




              35




              TikZ is awesome with a capital A. But load it by default? It takes up a lot of time and space. I would say only load it if you need it.
              – Matthew Leingang
              Nov 22 '10 at 12:53




              TikZ is awesome with a capital A. But load it by default? It takes up a lot of time and space. I would say only load it if you need it.
              – Matthew Leingang
              Nov 22 '10 at 12:53












              It takes time, but nearly always I find I need to do something with it.
              – fabikw
              Nov 23 '10 at 1:11




              It takes time, but nearly always I find I need to do something with it.
              – fabikw
              Nov 23 '10 at 1:11




              9




              9




              @levesque: Tikz has a fairly steep learning curve, but it is beautifully documented and provides rich libraries. I find the vector graphics that I produce in tikz to be superior to those I produced in inkscape. It seems easier on my brain to stay in keyboard mode as well.
              – philosodad
              Dec 29 '10 at 4:56




              @levesque: Tikz has a fairly steep learning curve, but it is beautifully documented and provides rich libraries. I find the vector graphics that I produce in tikz to be superior to those I produced in inkscape. It seems easier on my brain to stay in keyboard mode as well.
              – philosodad
              Dec 29 '10 at 4:56











              107














              Since my files nowadays has UTF-8 character encoding, I use this



              usepackage[utf8]{inputenc}





              share|improve this answer



















              • 37




                XeLaTeX or LuaLaTeX would be my choice for this
                – Joseph Wright
                Aug 15 '10 at 13:05










              • Interesting, must look into those projects.
                – Johan
                Aug 15 '10 at 13:16






              • 1




                Isn't it usepackage[utf8x]{inputenc}?
                – Olivier
                Jul 19 '11 at 8:17






              • 1




                I've experienced several cases where utf8x had a symbol that utf8 hadn't
                – Mog
                Nov 24 '12 at 11:47






              • 8




                @Olivier: utf8 is LaTeX base, while utf8x comes from the ucs package. So utf8 is portable.
                – Martin Schröder
                Jun 27 '13 at 14:39
















              107














              Since my files nowadays has UTF-8 character encoding, I use this



              usepackage[utf8]{inputenc}





              share|improve this answer



















              • 37




                XeLaTeX or LuaLaTeX would be my choice for this
                – Joseph Wright
                Aug 15 '10 at 13:05










              • Interesting, must look into those projects.
                – Johan
                Aug 15 '10 at 13:16






              • 1




                Isn't it usepackage[utf8x]{inputenc}?
                – Olivier
                Jul 19 '11 at 8:17






              • 1




                I've experienced several cases where utf8x had a symbol that utf8 hadn't
                – Mog
                Nov 24 '12 at 11:47






              • 8




                @Olivier: utf8 is LaTeX base, while utf8x comes from the ucs package. So utf8 is portable.
                – Martin Schröder
                Jun 27 '13 at 14:39














              107












              107








              107






              Since my files nowadays has UTF-8 character encoding, I use this



              usepackage[utf8]{inputenc}





              share|improve this answer














              Since my files nowadays has UTF-8 character encoding, I use this



              usepackage[utf8]{inputenc}






              share|improve this answer














              share|improve this answer



              share|improve this answer








              edited Jun 27 '13 at 17:35


























              community wiki





              4 revs, 2 users 75%
              Johan










              • 37




                XeLaTeX or LuaLaTeX would be my choice for this
                – Joseph Wright
                Aug 15 '10 at 13:05










              • Interesting, must look into those projects.
                – Johan
                Aug 15 '10 at 13:16






              • 1




                Isn't it usepackage[utf8x]{inputenc}?
                – Olivier
                Jul 19 '11 at 8:17






              • 1




                I've experienced several cases where utf8x had a symbol that utf8 hadn't
                – Mog
                Nov 24 '12 at 11:47






              • 8




                @Olivier: utf8 is LaTeX base, while utf8x comes from the ucs package. So utf8 is portable.
                – Martin Schröder
                Jun 27 '13 at 14:39














              • 37




                XeLaTeX or LuaLaTeX would be my choice for this
                – Joseph Wright
                Aug 15 '10 at 13:05










              • Interesting, must look into those projects.
                – Johan
                Aug 15 '10 at 13:16






              • 1




                Isn't it usepackage[utf8x]{inputenc}?
                – Olivier
                Jul 19 '11 at 8:17






              • 1




                I've experienced several cases where utf8x had a symbol that utf8 hadn't
                – Mog
                Nov 24 '12 at 11:47






              • 8




                @Olivier: utf8 is LaTeX base, while utf8x comes from the ucs package. So utf8 is portable.
                – Martin Schröder
                Jun 27 '13 at 14:39








              37




              37




              XeLaTeX or LuaLaTeX would be my choice for this
              – Joseph Wright
              Aug 15 '10 at 13:05




              XeLaTeX or LuaLaTeX would be my choice for this
              – Joseph Wright
              Aug 15 '10 at 13:05












              Interesting, must look into those projects.
              – Johan
              Aug 15 '10 at 13:16




              Interesting, must look into those projects.
              – Johan
              Aug 15 '10 at 13:16




              1




              1




              Isn't it usepackage[utf8x]{inputenc}?
              – Olivier
              Jul 19 '11 at 8:17




              Isn't it usepackage[utf8x]{inputenc}?
              – Olivier
              Jul 19 '11 at 8:17




              1




              1




              I've experienced several cases where utf8x had a symbol that utf8 hadn't
              – Mog
              Nov 24 '12 at 11:47




              I've experienced several cases where utf8x had a symbol that utf8 hadn't
              – Mog
              Nov 24 '12 at 11:47




              8




              8




              @Olivier: utf8 is LaTeX base, while utf8x comes from the ucs package. So utf8 is portable.
              – Martin Schröder
              Jun 27 '13 at 14:39




              @Olivier: utf8 is LaTeX base, while utf8x comes from the ucs package. So utf8 is portable.
              – Martin Schröder
              Jun 27 '13 at 14:39











              103














              usepackage{siunitx}


              siunitx, for typesetting units and especially for the "S" column type, which allows numbers in tables to be easily aligned, e.g. on the decimal marker.






              share|improve this answer



















              • 12




                usepackage[allowlitunits]{siunitx} is my normal incantation, it allows you to use things like 20millimeter directly in math mode.
                – Alex Hirzel
                May 1 '12 at 20:18












              • I have evidently never followed through on my plan to read the siunitx manual in depth. I was not aware of the S column type or allowlitunits, thank you!
                – owjburnham
                Jul 18 '17 at 9:50
















              103














              usepackage{siunitx}


              siunitx, for typesetting units and especially for the "S" column type, which allows numbers in tables to be easily aligned, e.g. on the decimal marker.






              share|improve this answer



















              • 12




                usepackage[allowlitunits]{siunitx} is my normal incantation, it allows you to use things like 20millimeter directly in math mode.
                – Alex Hirzel
                May 1 '12 at 20:18












              • I have evidently never followed through on my plan to read the siunitx manual in depth. I was not aware of the S column type or allowlitunits, thank you!
                – owjburnham
                Jul 18 '17 at 9:50














              103












              103








              103






              usepackage{siunitx}


              siunitx, for typesetting units and especially for the "S" column type, which allows numbers in tables to be easily aligned, e.g. on the decimal marker.






              share|improve this answer














              usepackage{siunitx}


              siunitx, for typesetting units and especially for the "S" column type, which allows numbers in tables to be easily aligned, e.g. on the decimal marker.







              share|improve this answer














              share|improve this answer



              share|improve this answer








              edited Jun 4 '12 at 16:20


























              community wiki





              2 revs, 2 users 67%
              Jake










              • 12




                usepackage[allowlitunits]{siunitx} is my normal incantation, it allows you to use things like 20millimeter directly in math mode.
                – Alex Hirzel
                May 1 '12 at 20:18












              • I have evidently never followed through on my plan to read the siunitx manual in depth. I was not aware of the S column type or allowlitunits, thank you!
                – owjburnham
                Jul 18 '17 at 9:50














              • 12




                usepackage[allowlitunits]{siunitx} is my normal incantation, it allows you to use things like 20millimeter directly in math mode.
                – Alex Hirzel
                May 1 '12 at 20:18












              • I have evidently never followed through on my plan to read the siunitx manual in depth. I was not aware of the S column type or allowlitunits, thank you!
                – owjburnham
                Jul 18 '17 at 9:50








              12




              12




              usepackage[allowlitunits]{siunitx} is my normal incantation, it allows you to use things like 20millimeter directly in math mode.
              – Alex Hirzel
              May 1 '12 at 20:18






              usepackage[allowlitunits]{siunitx} is my normal incantation, it allows you to use things like 20millimeter directly in math mode.
              – Alex Hirzel
              May 1 '12 at 20:18














              I have evidently never followed through on my plan to read the siunitx manual in depth. I was not aware of the S column type or allowlitunits, thank you!
              – owjburnham
              Jul 18 '17 at 9:50




              I have evidently never followed through on my plan to read the siunitx manual in depth. I was not aware of the S column type or allowlitunits, thank you!
              – owjburnham
              Jul 18 '17 at 9:50











              79














              The 'rich' document classes such as memoir and KOMA-Script include a lot of functionality that is not available from the LaTeX kernel. So the packages you load when using the article class might be rather different from those when using memoir. A lot of packages that get used by many people with the base classes (things like float, caption, tocbibind and titlesec) are covered by the richer document classes.






              share|improve this answer



















              • 16




                begin{gripe} My problems with these richer document classes are that it makes it very difficult to pick and choose, and that it is a major pain when Big Shot Journal says "please rewrite your document to use our class file" (there's even a journal that won't let you send an accompanying style file). end{gripe}
                – Loop Space
                Jul 29 '10 at 13:19






              • 10




                I tend to stick to article + packages, myself, so I can sympathise. All the more reason for me to get on and get LaTeX3 finished, so we can have a good set of abilities out of the box!
                – Joseph Wright
                Jul 29 '10 at 14:33






              • 9




                begin{joke} Then stop wasting time here and get on with it! end{joke}
                – Loop Space
                Jul 29 '10 at 18:11








              • 3




                If only it were that easy :-) If you want to see that things are happening, there is an RSS feed for SVN checkins: latex-project.org/latex3svn.rss
                – Joseph Wright
                Jul 29 '10 at 21:36






              • 13




                That gripe seems a gripe with the journals, rather than with the rich document classes. Also, if you're writing a journal article, memoir doesn't seem like an obvious way to go, if you are going to end up having to conform to some journal's style eventually. Again, that's not an issue with rich document classes, that's just a matter of picking the right tool for the job. And for journal submissions, minimal package requirements and basic document classes seems a good modus operandi
                – Seamus
                Aug 1 '10 at 10:41
















              79














              The 'rich' document classes such as memoir and KOMA-Script include a lot of functionality that is not available from the LaTeX kernel. So the packages you load when using the article class might be rather different from those when using memoir. A lot of packages that get used by many people with the base classes (things like float, caption, tocbibind and titlesec) are covered by the richer document classes.






              share|improve this answer



















              • 16




                begin{gripe} My problems with these richer document classes are that it makes it very difficult to pick and choose, and that it is a major pain when Big Shot Journal says "please rewrite your document to use our class file" (there's even a journal that won't let you send an accompanying style file). end{gripe}
                – Loop Space
                Jul 29 '10 at 13:19






              • 10




                I tend to stick to article + packages, myself, so I can sympathise. All the more reason for me to get on and get LaTeX3 finished, so we can have a good set of abilities out of the box!
                – Joseph Wright
                Jul 29 '10 at 14:33






              • 9




                begin{joke} Then stop wasting time here and get on with it! end{joke}
                – Loop Space
                Jul 29 '10 at 18:11








              • 3




                If only it were that easy :-) If you want to see that things are happening, there is an RSS feed for SVN checkins: latex-project.org/latex3svn.rss
                – Joseph Wright
                Jul 29 '10 at 21:36






              • 13




                That gripe seems a gripe with the journals, rather than with the rich document classes. Also, if you're writing a journal article, memoir doesn't seem like an obvious way to go, if you are going to end up having to conform to some journal's style eventually. Again, that's not an issue with rich document classes, that's just a matter of picking the right tool for the job. And for journal submissions, minimal package requirements and basic document classes seems a good modus operandi
                – Seamus
                Aug 1 '10 at 10:41














              79












              79








              79






              The 'rich' document classes such as memoir and KOMA-Script include a lot of functionality that is not available from the LaTeX kernel. So the packages you load when using the article class might be rather different from those when using memoir. A lot of packages that get used by many people with the base classes (things like float, caption, tocbibind and titlesec) are covered by the richer document classes.






              share|improve this answer














              The 'rich' document classes such as memoir and KOMA-Script include a lot of functionality that is not available from the LaTeX kernel. So the packages you load when using the article class might be rather different from those when using memoir. A lot of packages that get used by many people with the base classes (things like float, caption, tocbibind and titlesec) are covered by the richer document classes.







              share|improve this answer














              share|improve this answer



              share|improve this answer








              edited Jul 31 '10 at 4:33


























              community wiki





              3 revs, 3 users 50%
              Joseph Wright










              • 16




                begin{gripe} My problems with these richer document classes are that it makes it very difficult to pick and choose, and that it is a major pain when Big Shot Journal says "please rewrite your document to use our class file" (there's even a journal that won't let you send an accompanying style file). end{gripe}
                – Loop Space
                Jul 29 '10 at 13:19






              • 10




                I tend to stick to article + packages, myself, so I can sympathise. All the more reason for me to get on and get LaTeX3 finished, so we can have a good set of abilities out of the box!
                – Joseph Wright
                Jul 29 '10 at 14:33






              • 9




                begin{joke} Then stop wasting time here and get on with it! end{joke}
                – Loop Space
                Jul 29 '10 at 18:11








              • 3




                If only it were that easy :-) If you want to see that things are happening, there is an RSS feed for SVN checkins: latex-project.org/latex3svn.rss
                – Joseph Wright
                Jul 29 '10 at 21:36






              • 13




                That gripe seems a gripe with the journals, rather than with the rich document classes. Also, if you're writing a journal article, memoir doesn't seem like an obvious way to go, if you are going to end up having to conform to some journal's style eventually. Again, that's not an issue with rich document classes, that's just a matter of picking the right tool for the job. And for journal submissions, minimal package requirements and basic document classes seems a good modus operandi
                – Seamus
                Aug 1 '10 at 10:41














              • 16




                begin{gripe} My problems with these richer document classes are that it makes it very difficult to pick and choose, and that it is a major pain when Big Shot Journal says "please rewrite your document to use our class file" (there's even a journal that won't let you send an accompanying style file). end{gripe}
                – Loop Space
                Jul 29 '10 at 13:19






              • 10




                I tend to stick to article + packages, myself, so I can sympathise. All the more reason for me to get on and get LaTeX3 finished, so we can have a good set of abilities out of the box!
                – Joseph Wright
                Jul 29 '10 at 14:33






              • 9




                begin{joke} Then stop wasting time here and get on with it! end{joke}
                – Loop Space
                Jul 29 '10 at 18:11








              • 3




                If only it were that easy :-) If you want to see that things are happening, there is an RSS feed for SVN checkins: latex-project.org/latex3svn.rss
                – Joseph Wright
                Jul 29 '10 at 21:36






              • 13




                That gripe seems a gripe with the journals, rather than with the rich document classes. Also, if you're writing a journal article, memoir doesn't seem like an obvious way to go, if you are going to end up having to conform to some journal's style eventually. Again, that's not an issue with rich document classes, that's just a matter of picking the right tool for the job. And for journal submissions, minimal package requirements and basic document classes seems a good modus operandi
                – Seamus
                Aug 1 '10 at 10:41








              16




              16




              begin{gripe} My problems with these richer document classes are that it makes it very difficult to pick and choose, and that it is a major pain when Big Shot Journal says "please rewrite your document to use our class file" (there's even a journal that won't let you send an accompanying style file). end{gripe}
              – Loop Space
              Jul 29 '10 at 13:19




              begin{gripe} My problems with these richer document classes are that it makes it very difficult to pick and choose, and that it is a major pain when Big Shot Journal says "please rewrite your document to use our class file" (there's even a journal that won't let you send an accompanying style file). end{gripe}
              – Loop Space
              Jul 29 '10 at 13:19




              10




              10




              I tend to stick to article + packages, myself, so I can sympathise. All the more reason for me to get on and get LaTeX3 finished, so we can have a good set of abilities out of the box!
              – Joseph Wright
              Jul 29 '10 at 14:33




              I tend to stick to article + packages, myself, so I can sympathise. All the more reason for me to get on and get LaTeX3 finished, so we can have a good set of abilities out of the box!
              – Joseph Wright
              Jul 29 '10 at 14:33




              9




              9




              begin{joke} Then stop wasting time here and get on with it! end{joke}
              – Loop Space
              Jul 29 '10 at 18:11






              begin{joke} Then stop wasting time here and get on with it! end{joke}
              – Loop Space
              Jul 29 '10 at 18:11






              3




              3




              If only it were that easy :-) If you want to see that things are happening, there is an RSS feed for SVN checkins: latex-project.org/latex3svn.rss
              – Joseph Wright
              Jul 29 '10 at 21:36




              If only it were that easy :-) If you want to see that things are happening, there is an RSS feed for SVN checkins: latex-project.org/latex3svn.rss
              – Joseph Wright
              Jul 29 '10 at 21:36




              13




              13




              That gripe seems a gripe with the journals, rather than with the rich document classes. Also, if you're writing a journal article, memoir doesn't seem like an obvious way to go, if you are going to end up having to conform to some journal's style eventually. Again, that's not an issue with rich document classes, that's just a matter of picking the right tool for the job. And for journal submissions, minimal package requirements and basic document classes seems a good modus operandi
              – Seamus
              Aug 1 '10 at 10:41




              That gripe seems a gripe with the journals, rather than with the rich document classes. Also, if you're writing a journal article, memoir doesn't seem like an obvious way to go, if you are going to end up having to conform to some journal's style eventually. Again, that's not an issue with rich document classes, that's just a matter of picking the right tool for the job. And for journal submissions, minimal package requirements and basic document classes seems a good modus operandi
              – Seamus
              Aug 1 '10 at 10:41











              72














              usepackage{graphicx}


              For including figures, rotating or scaling text. I also use the graphicspath command to specify a subfolder to help organize my figures and so I can easily change between, for example, a set of figures for internal used (with extra info) and final versions for distribution.






              share|improve this answer




























                72














                usepackage{graphicx}


                For including figures, rotating or scaling text. I also use the graphicspath command to specify a subfolder to help organize my figures and so I can easily change between, for example, a set of figures for internal used (with extra info) and final versions for distribution.






                share|improve this answer


























                  72












                  72








                  72






                  usepackage{graphicx}


                  For including figures, rotating or scaling text. I also use the graphicspath command to specify a subfolder to help organize my figures and so I can easily change between, for example, a set of figures for internal used (with extra info) and final versions for distribution.






                  share|improve this answer














                  usepackage{graphicx}


                  For including figures, rotating or scaling text. I also use the graphicspath command to specify a subfolder to help organize my figures and so I can easily change between, for example, a set of figures for internal used (with extra info) and final versions for distribution.







                  share|improve this answer














                  share|improve this answer



                  share|improve this answer








                  answered May 1 '12 at 19:19


























                  community wiki





                  mforbes
























                      63














                      usepackage{lmodern}  % better i18n Postscript version of Knuth's cm fonts





                      share|improve this answer























                      • related question: tex.stackexchange.com/questions/1390/latin-modern-vs-cm-super
                        – matth
                        Jun 26 '12 at 21:24






                      • 1




                        I never use this except for debugging. usepackage{cfr-lm} ...
                        – cfr
                        Nov 24 '16 at 0:58
















                      63














                      usepackage{lmodern}  % better i18n Postscript version of Knuth's cm fonts





                      share|improve this answer























                      • related question: tex.stackexchange.com/questions/1390/latin-modern-vs-cm-super
                        – matth
                        Jun 26 '12 at 21:24






                      • 1




                        I never use this except for debugging. usepackage{cfr-lm} ...
                        – cfr
                        Nov 24 '16 at 0:58














                      63












                      63








                      63






                      usepackage{lmodern}  % better i18n Postscript version of Knuth's cm fonts





                      share|improve this answer














                      usepackage{lmodern}  % better i18n Postscript version of Knuth's cm fonts






                      share|improve this answer














                      share|improve this answer



                      share|improve this answer








                      edited Dec 28 '10 at 6:29


























                      community wiki





                      2 revs, 2 users 67%
                      towolf














                      • related question: tex.stackexchange.com/questions/1390/latin-modern-vs-cm-super
                        – matth
                        Jun 26 '12 at 21:24






                      • 1




                        I never use this except for debugging. usepackage{cfr-lm} ...
                        – cfr
                        Nov 24 '16 at 0:58


















                      • related question: tex.stackexchange.com/questions/1390/latin-modern-vs-cm-super
                        – matth
                        Jun 26 '12 at 21:24






                      • 1




                        I never use this except for debugging. usepackage{cfr-lm} ...
                        – cfr
                        Nov 24 '16 at 0:58
















                      related question: tex.stackexchange.com/questions/1390/latin-modern-vs-cm-super
                      – matth
                      Jun 26 '12 at 21:24




                      related question: tex.stackexchange.com/questions/1390/latin-modern-vs-cm-super
                      – matth
                      Jun 26 '12 at 21:24




                      1




                      1




                      I never use this except for debugging. usepackage{cfr-lm} ...
                      – cfr
                      Nov 24 '16 at 0:58




                      I never use this except for debugging. usepackage{cfr-lm} ...
                      – cfr
                      Nov 24 '16 at 0:58











                      61














                      In addition to many packages already listed here, I always include mathtools. It provides implementations of mathclap (and similar commands) as well as nice extensible arrow.






                      share|improve this answer



















                      • 4




                        mathclap is great. I use it to great effect for things like sum_{mathclap{big long thing}}. (It's also amusingly named with at least one off-color meaning.)
                        – TH.
                        Aug 27 '10 at 9:36






                      • 2




                        shortintertext is also provided by the mathtools package and provids tighter vertical spacing compared to intertext from the amsmath package.
                        – Peter Grill
                        May 2 '12 at 0:47
















                      61














                      In addition to many packages already listed here, I always include mathtools. It provides implementations of mathclap (and similar commands) as well as nice extensible arrow.






                      share|improve this answer



















                      • 4




                        mathclap is great. I use it to great effect for things like sum_{mathclap{big long thing}}. (It's also amusingly named with at least one off-color meaning.)
                        – TH.
                        Aug 27 '10 at 9:36






                      • 2




                        shortintertext is also provided by the mathtools package and provids tighter vertical spacing compared to intertext from the amsmath package.
                        – Peter Grill
                        May 2 '12 at 0:47














                      61












                      61








                      61






                      In addition to many packages already listed here, I always include mathtools. It provides implementations of mathclap (and similar commands) as well as nice extensible arrow.






                      share|improve this answer














                      In addition to many packages already listed here, I always include mathtools. It provides implementations of mathclap (and similar commands) as well as nice extensible arrow.







                      share|improve this answer














                      share|improve this answer



                      share|improve this answer








                      answered Jul 31 '10 at 11:24


























                      community wiki





                      Caramdir









                      • 4




                        mathclap is great. I use it to great effect for things like sum_{mathclap{big long thing}}. (It's also amusingly named with at least one off-color meaning.)
                        – TH.
                        Aug 27 '10 at 9:36






                      • 2




                        shortintertext is also provided by the mathtools package and provids tighter vertical spacing compared to intertext from the amsmath package.
                        – Peter Grill
                        May 2 '12 at 0:47














                      • 4




                        mathclap is great. I use it to great effect for things like sum_{mathclap{big long thing}}. (It's also amusingly named with at least one off-color meaning.)
                        – TH.
                        Aug 27 '10 at 9:36






                      • 2




                        shortintertext is also provided by the mathtools package and provids tighter vertical spacing compared to intertext from the amsmath package.
                        – Peter Grill
                        May 2 '12 at 0:47








                      4




                      4




                      mathclap is great. I use it to great effect for things like sum_{mathclap{big long thing}}. (It's also amusingly named with at least one off-color meaning.)
                      – TH.
                      Aug 27 '10 at 9:36




                      mathclap is great. I use it to great effect for things like sum_{mathclap{big long thing}}. (It's also amusingly named with at least one off-color meaning.)
                      – TH.
                      Aug 27 '10 at 9:36




                      2




                      2




                      shortintertext is also provided by the mathtools package and provids tighter vertical spacing compared to intertext from the amsmath package.
                      – Peter Grill
                      May 2 '12 at 0:47




                      shortintertext is also provided by the mathtools package and provids tighter vertical spacing compared to intertext from the amsmath package.
                      – Peter Grill
                      May 2 '12 at 0:47











                      60














                      I can't live without listings --- pretty-printing (colours, formatting and all) algorithms and code is indispensable --- in pretty much any programming languages and dialects under the sun. Plus, I can import a source file directly from the repository, and the latest version will be automatically rendered.






                      share|improve this answer



















                      • 1




                        I was pleasantly surprised that I could prettyprint MIPS assembly language code with listings! Excellent package.
                        – MercurialMadnessMan
                        Nov 24 '12 at 6:30
















                      60














                      I can't live without listings --- pretty-printing (colours, formatting and all) algorithms and code is indispensable --- in pretty much any programming languages and dialects under the sun. Plus, I can import a source file directly from the repository, and the latest version will be automatically rendered.






                      share|improve this answer



















                      • 1




                        I was pleasantly surprised that I could prettyprint MIPS assembly language code with listings! Excellent package.
                        – MercurialMadnessMan
                        Nov 24 '12 at 6:30














                      60












                      60








                      60






                      I can't live without listings --- pretty-printing (colours, formatting and all) algorithms and code is indispensable --- in pretty much any programming languages and dialects under the sun. Plus, I can import a source file directly from the repository, and the latest version will be automatically rendered.






                      share|improve this answer














                      I can't live without listings --- pretty-printing (colours, formatting and all) algorithms and code is indispensable --- in pretty much any programming languages and dialects under the sun. Plus, I can import a source file directly from the repository, and the latest version will be automatically rendered.







                      share|improve this answer














                      share|improve this answer



                      share|improve this answer








                      edited Jun 4 '12 at 16:26


























                      community wiki





                      2 revs, 2 users 67%
                      Martin Tapankov










                      • 1




                        I was pleasantly surprised that I could prettyprint MIPS assembly language code with listings! Excellent package.
                        – MercurialMadnessMan
                        Nov 24 '12 at 6:30














                      • 1




                        I was pleasantly surprised that I could prettyprint MIPS assembly language code with listings! Excellent package.
                        – MercurialMadnessMan
                        Nov 24 '12 at 6:30








                      1




                      1




                      I was pleasantly surprised that I could prettyprint MIPS assembly language code with listings! Excellent package.
                      – MercurialMadnessMan
                      Nov 24 '12 at 6:30




                      I was pleasantly surprised that I could prettyprint MIPS assembly language code with listings! Excellent package.
                      – MercurialMadnessMan
                      Nov 24 '12 at 6:30











                      56














                      The package xspace lets you define commands that don't eat up whitespace after them. So you can define an abbreviation like



                      newcommand{sA}{mathcal{A}xspace}



                      and then you can type objects of sA are called widgets instead of objects of sA are called widgets.






                      share|improve this answer



















                      • 1




                        That's one I use so much that I forget it's not part of the main code!
                        – Loop Space
                        Aug 5 '10 at 7:10






                      • 20




                        On comp.text.tex there's a series of messages "xspace and italic correction" about spacing inconsistencies created by xspace. There, Will Robertson suggested "delimited macros" as an alternative to xspace. Using newcommand* only to ensure that no existing command is overriden, the above example would look like this: newcommand*{sA}{}defsA/{mathcal{A}} To quote Will Robertson: "In the source you must always type "foo/" [here: "sA/"] (or TeX will throw an error), and spaces after it won't be gobbled."
                        – lockstep
                        Aug 6 '10 at 15:04






                      • 6




                        The main advantage of sa/ is that an error message will occur if you happen to forget the closing slash. On the contrary, if you happen to forget the closing backslash of sA , you'll end with gobbled space without noticing it.
                        – lockstep
                        Aug 11 '10 at 20:50








                      • 6




                        I used xspace one time in a paper with other authors. It was a huge pain since some macros didn't behave like others. It led to all sort of confusion, especially when thinks like foo bar no long work as you expect because foo's definition ends with xspace. I've never tried foo/. The main advantage I see with that is if your macro is m/...
                        – TH.
                        Aug 27 '10 at 9:32






                      • 9




                        I don't especially like the look of sA/ but I can't think of a better delimiter to use. Perhaps a semicolon would be fine (after HTML): sA;. My personal belief is that non-delimited macros without arguments (i.e., the ones that gobble spaces) are just plain wrong for document commands because of the spacing problems. Even experienced LaTeX authors trip up with them.
                        – Will Robertson
                        Sep 2 '10 at 9:28
















                      56














                      The package xspace lets you define commands that don't eat up whitespace after them. So you can define an abbreviation like



                      newcommand{sA}{mathcal{A}xspace}



                      and then you can type objects of sA are called widgets instead of objects of sA are called widgets.






                      share|improve this answer



















                      • 1




                        That's one I use so much that I forget it's not part of the main code!
                        – Loop Space
                        Aug 5 '10 at 7:10






                      • 20




                        On comp.text.tex there's a series of messages "xspace and italic correction" about spacing inconsistencies created by xspace. There, Will Robertson suggested "delimited macros" as an alternative to xspace. Using newcommand* only to ensure that no existing command is overriden, the above example would look like this: newcommand*{sA}{}defsA/{mathcal{A}} To quote Will Robertson: "In the source you must always type "foo/" [here: "sA/"] (or TeX will throw an error), and spaces after it won't be gobbled."
                        – lockstep
                        Aug 6 '10 at 15:04






                      • 6




                        The main advantage of sa/ is that an error message will occur if you happen to forget the closing slash. On the contrary, if you happen to forget the closing backslash of sA , you'll end with gobbled space without noticing it.
                        – lockstep
                        Aug 11 '10 at 20:50








                      • 6




                        I used xspace one time in a paper with other authors. It was a huge pain since some macros didn't behave like others. It led to all sort of confusion, especially when thinks like foo bar no long work as you expect because foo's definition ends with xspace. I've never tried foo/. The main advantage I see with that is if your macro is m/...
                        – TH.
                        Aug 27 '10 at 9:32






                      • 9




                        I don't especially like the look of sA/ but I can't think of a better delimiter to use. Perhaps a semicolon would be fine (after HTML): sA;. My personal belief is that non-delimited macros without arguments (i.e., the ones that gobble spaces) are just plain wrong for document commands because of the spacing problems. Even experienced LaTeX authors trip up with them.
                        – Will Robertson
                        Sep 2 '10 at 9:28














                      56












                      56








                      56






                      The package xspace lets you define commands that don't eat up whitespace after them. So you can define an abbreviation like



                      newcommand{sA}{mathcal{A}xspace}



                      and then you can type objects of sA are called widgets instead of objects of sA are called widgets.






                      share|improve this answer














                      The package xspace lets you define commands that don't eat up whitespace after them. So you can define an abbreviation like



                      newcommand{sA}{mathcal{A}xspace}



                      and then you can type objects of sA are called widgets instead of objects of sA are called widgets.







                      share|improve this answer














                      share|improve this answer



                      share|improve this answer








                      edited Jun 4 '12 at 16:23


























                      community wiki





                      2 revs, 2 users 83%
                      Mike Shulman










                      • 1




                        That's one I use so much that I forget it's not part of the main code!
                        – Loop Space
                        Aug 5 '10 at 7:10






                      • 20




                        On comp.text.tex there's a series of messages "xspace and italic correction" about spacing inconsistencies created by xspace. There, Will Robertson suggested "delimited macros" as an alternative to xspace. Using newcommand* only to ensure that no existing command is overriden, the above example would look like this: newcommand*{sA}{}defsA/{mathcal{A}} To quote Will Robertson: "In the source you must always type "foo/" [here: "sA/"] (or TeX will throw an error), and spaces after it won't be gobbled."
                        – lockstep
                        Aug 6 '10 at 15:04






                      • 6




                        The main advantage of sa/ is that an error message will occur if you happen to forget the closing slash. On the contrary, if you happen to forget the closing backslash of sA , you'll end with gobbled space without noticing it.
                        – lockstep
                        Aug 11 '10 at 20:50








                      • 6




                        I used xspace one time in a paper with other authors. It was a huge pain since some macros didn't behave like others. It led to all sort of confusion, especially when thinks like foo bar no long work as you expect because foo's definition ends with xspace. I've never tried foo/. The main advantage I see with that is if your macro is m/...
                        – TH.
                        Aug 27 '10 at 9:32






                      • 9




                        I don't especially like the look of sA/ but I can't think of a better delimiter to use. Perhaps a semicolon would be fine (after HTML): sA;. My personal belief is that non-delimited macros without arguments (i.e., the ones that gobble spaces) are just plain wrong for document commands because of the spacing problems. Even experienced LaTeX authors trip up with them.
                        – Will Robertson
                        Sep 2 '10 at 9:28














                      • 1




                        That's one I use so much that I forget it's not part of the main code!
                        – Loop Space
                        Aug 5 '10 at 7:10






                      • 20




                        On comp.text.tex there's a series of messages "xspace and italic correction" about spacing inconsistencies created by xspace. There, Will Robertson suggested "delimited macros" as an alternative to xspace. Using newcommand* only to ensure that no existing command is overriden, the above example would look like this: newcommand*{sA}{}defsA/{mathcal{A}} To quote Will Robertson: "In the source you must always type "foo/" [here: "sA/"] (or TeX will throw an error), and spaces after it won't be gobbled."
                        – lockstep
                        Aug 6 '10 at 15:04






                      • 6




                        The main advantage of sa/ is that an error message will occur if you happen to forget the closing slash. On the contrary, if you happen to forget the closing backslash of sA , you'll end with gobbled space without noticing it.
                        – lockstep
                        Aug 11 '10 at 20:50








                      • 6




                        I used xspace one time in a paper with other authors. It was a huge pain since some macros didn't behave like others. It led to all sort of confusion, especially when thinks like foo bar no long work as you expect because foo's definition ends with xspace. I've never tried foo/. The main advantage I see with that is if your macro is m/...
                        – TH.
                        Aug 27 '10 at 9:32






                      • 9




                        I don't especially like the look of sA/ but I can't think of a better delimiter to use. Perhaps a semicolon would be fine (after HTML): sA;. My personal belief is that non-delimited macros without arguments (i.e., the ones that gobble spaces) are just plain wrong for document commands because of the spacing problems. Even experienced LaTeX authors trip up with them.
                        – Will Robertson
                        Sep 2 '10 at 9:28








                      1




                      1




                      That's one I use so much that I forget it's not part of the main code!
                      – Loop Space
                      Aug 5 '10 at 7:10




                      That's one I use so much that I forget it's not part of the main code!
                      – Loop Space
                      Aug 5 '10 at 7:10




                      20




                      20




                      On comp.text.tex there's a series of messages "xspace and italic correction" about spacing inconsistencies created by xspace. There, Will Robertson suggested "delimited macros" as an alternative to xspace. Using newcommand* only to ensure that no existing command is overriden, the above example would look like this: newcommand*{sA}{}defsA/{mathcal{A}} To quote Will Robertson: "In the source you must always type "foo/" [here: "sA/"] (or TeX will throw an error), and spaces after it won't be gobbled."
                      – lockstep
                      Aug 6 '10 at 15:04




                      On comp.text.tex there's a series of messages "xspace and italic correction" about spacing inconsistencies created by xspace. There, Will Robertson suggested "delimited macros" as an alternative to xspace. Using newcommand* only to ensure that no existing command is overriden, the above example would look like this: newcommand*{sA}{}defsA/{mathcal{A}} To quote Will Robertson: "In the source you must always type "foo/" [here: "sA/"] (or TeX will throw an error), and spaces after it won't be gobbled."
                      – lockstep
                      Aug 6 '10 at 15:04




                      6




                      6




                      The main advantage of sa/ is that an error message will occur if you happen to forget the closing slash. On the contrary, if you happen to forget the closing backslash of sA , you'll end with gobbled space without noticing it.
                      – lockstep
                      Aug 11 '10 at 20:50






                      The main advantage of sa/ is that an error message will occur if you happen to forget the closing slash. On the contrary, if you happen to forget the closing backslash of sA , you'll end with gobbled space without noticing it.
                      – lockstep
                      Aug 11 '10 at 20:50






                      6




                      6




                      I used xspace one time in a paper with other authors. It was a huge pain since some macros didn't behave like others. It led to all sort of confusion, especially when thinks like foo bar no long work as you expect because foo's definition ends with xspace. I've never tried foo/. The main advantage I see with that is if your macro is m/...
                      – TH.
                      Aug 27 '10 at 9:32




                      I used xspace one time in a paper with other authors. It was a huge pain since some macros didn't behave like others. It led to all sort of confusion, especially when thinks like foo bar no long work as you expect because foo's definition ends with xspace. I've never tried foo/. The main advantage I see with that is if your macro is m/...
                      – TH.
                      Aug 27 '10 at 9:32




                      9




                      9




                      I don't especially like the look of sA/ but I can't think of a better delimiter to use. Perhaps a semicolon would be fine (after HTML): sA;. My personal belief is that non-delimited macros without arguments (i.e., the ones that gobble spaces) are just plain wrong for document commands because of the spacing problems. Even experienced LaTeX authors trip up with them.
                      – Will Robertson
                      Sep 2 '10 at 9:28




                      I don't especially like the look of sA/ but I can't think of a better delimiter to use. Perhaps a semicolon would be fine (after HTML): sA;. My personal belief is that non-delimited macros without arguments (i.e., the ones that gobble spaces) are just plain wrong for document commands because of the spacing problems. Even experienced LaTeX authors trip up with them.
                      – Will Robertson
                      Sep 2 '10 at 9:28











                      52














                      First line of the document should be



                      RequirePackage{fixltx2e}
                      documentclass{...}


                      , which fixes a few things in the LaTeX2e kernel.



                      Due to LaTeX's stability policy, these corrections have not been incorporated into the LaTeX2e kernel, but this package does things most people would agree are bugfixes. So to load this package is always recommended for newly created documents. The corrections have no commonalities, but the package's description has a nice summary:





                      • ensure one-column floats don't get ahead of two-column floats;

                      • correct page headers in twocolumn documents;

                      • stop spaces disappearing in moving arguments;

                      • allowing fnsymbol to use text symbols;

                      • allow the first word after a float to hyphenate;


                      • emph can produce caps/small caps text;

                      • bugs in setlength and flushbottom.






                      EDIT 27.01.2016:



                      This package is obsolete for LaTeX releases after 2015. See latexrelease.pdf.






                      share|improve this answer



















                      • 1




                        It should be RequirePackage{fixltx2e} as first line of you'require document, even before the document class, see texdev.net/2014/12/28/fixing-latex2e
                        – MaxNoe
                        Jan 17 '15 at 13:51






                      • 1




                        really should be an argument to documentclass.
                        – ivo Welch
                        Jan 2 '16 at 16:58






                      • 7




                        fixltx2e is not required with releases after 2015(fixltx2e) All fixes are now in the LaTeX kernel.
                        – kaka
                        Apr 3 '16 at 11:23
















                      52














                      First line of the document should be



                      RequirePackage{fixltx2e}
                      documentclass{...}


                      , which fixes a few things in the LaTeX2e kernel.



                      Due to LaTeX's stability policy, these corrections have not been incorporated into the LaTeX2e kernel, but this package does things most people would agree are bugfixes. So to load this package is always recommended for newly created documents. The corrections have no commonalities, but the package's description has a nice summary:





                      • ensure one-column floats don't get ahead of two-column floats;

                      • correct page headers in twocolumn documents;

                      • stop spaces disappearing in moving arguments;

                      • allowing fnsymbol to use text symbols;

                      • allow the first word after a float to hyphenate;


                      • emph can produce caps/small caps text;

                      • bugs in setlength and flushbottom.






                      EDIT 27.01.2016:



                      This package is obsolete for LaTeX releases after 2015. See latexrelease.pdf.






                      share|improve this answer



















                      • 1




                        It should be RequirePackage{fixltx2e} as first line of you'require document, even before the document class, see texdev.net/2014/12/28/fixing-latex2e
                        – MaxNoe
                        Jan 17 '15 at 13:51






                      • 1




                        really should be an argument to documentclass.
                        – ivo Welch
                        Jan 2 '16 at 16:58






                      • 7




                        fixltx2e is not required with releases after 2015(fixltx2e) All fixes are now in the LaTeX kernel.
                        – kaka
                        Apr 3 '16 at 11:23














                      52












                      52








                      52






                      First line of the document should be



                      RequirePackage{fixltx2e}
                      documentclass{...}


                      , which fixes a few things in the LaTeX2e kernel.



                      Due to LaTeX's stability policy, these corrections have not been incorporated into the LaTeX2e kernel, but this package does things most people would agree are bugfixes. So to load this package is always recommended for newly created documents. The corrections have no commonalities, but the package's description has a nice summary:





                      • ensure one-column floats don't get ahead of two-column floats;

                      • correct page headers in twocolumn documents;

                      • stop spaces disappearing in moving arguments;

                      • allowing fnsymbol to use text symbols;

                      • allow the first word after a float to hyphenate;


                      • emph can produce caps/small caps text;

                      • bugs in setlength and flushbottom.






                      EDIT 27.01.2016:



                      This package is obsolete for LaTeX releases after 2015. See latexrelease.pdf.






                      share|improve this answer














                      First line of the document should be



                      RequirePackage{fixltx2e}
                      documentclass{...}


                      , which fixes a few things in the LaTeX2e kernel.



                      Due to LaTeX's stability policy, these corrections have not been incorporated into the LaTeX2e kernel, but this package does things most people would agree are bugfixes. So to load this package is always recommended for newly created documents. The corrections have no commonalities, but the package's description has a nice summary:





                      • ensure one-column floats don't get ahead of two-column floats;

                      • correct page headers in twocolumn documents;

                      • stop spaces disappearing in moving arguments;

                      • allowing fnsymbol to use text symbols;

                      • allow the first word after a float to hyphenate;


                      • emph can produce caps/small caps text;

                      • bugs in setlength and flushbottom.






                      EDIT 27.01.2016:



                      This package is obsolete for LaTeX releases after 2015. See latexrelease.pdf.







                      share|improve this answer














                      share|improve this answer



                      share|improve this answer








                      edited Jan 27 '16 at 13:41


























                      community wiki





                      11 revs, 5 users 36%
                      doncherry









                      • 1




                        It should be RequirePackage{fixltx2e} as first line of you'require document, even before the document class, see texdev.net/2014/12/28/fixing-latex2e
                        – MaxNoe
                        Jan 17 '15 at 13:51






                      • 1




                        really should be an argument to documentclass.
                        – ivo Welch
                        Jan 2 '16 at 16:58






                      • 7




                        fixltx2e is not required with releases after 2015(fixltx2e) All fixes are now in the LaTeX kernel.
                        – kaka
                        Apr 3 '16 at 11:23














                      • 1




                        It should be RequirePackage{fixltx2e} as first line of you'require document, even before the document class, see texdev.net/2014/12/28/fixing-latex2e
                        – MaxNoe
                        Jan 17 '15 at 13:51






                      • 1




                        really should be an argument to documentclass.
                        – ivo Welch
                        Jan 2 '16 at 16:58






                      • 7




                        fixltx2e is not required with releases after 2015(fixltx2e) All fixes are now in the LaTeX kernel.
                        – kaka
                        Apr 3 '16 at 11:23








                      1




                      1




                      It should be RequirePackage{fixltx2e} as first line of you'require document, even before the document class, see texdev.net/2014/12/28/fixing-latex2e
                      – MaxNoe
                      Jan 17 '15 at 13:51




                      It should be RequirePackage{fixltx2e} as first line of you'require document, even before the document class, see texdev.net/2014/12/28/fixing-latex2e
                      – MaxNoe
                      Jan 17 '15 at 13:51




                      1




                      1




                      really should be an argument to documentclass.
                      – ivo Welch
                      Jan 2 '16 at 16:58




                      really should be an argument to documentclass.
                      – ivo Welch
                      Jan 2 '16 at 16:58




                      7




                      7




                      fixltx2e is not required with releases after 2015(fixltx2e) All fixes are now in the LaTeX kernel.
                      – kaka
                      Apr 3 '16 at 11:23




                      fixltx2e is not required with releases after 2015(fixltx2e) All fixes are now in the LaTeX kernel.
                      – kaka
                      Apr 3 '16 at 11:23











                      51














                      For papers on the arXiv (maths, physics and computer science mostly) there's a list of packages sorted by frequency of use.



                      The top twenty packages are:




                      1. article

                      2. graphicx

                      3. amssymb

                      4. amsmath

                      5. revtex

                      6. revtex4

                      7. epsfig

                      8. amsfonts

                      9. bm

                      10. latexsym

                      11. amsart

                      12. dcolumn

                      13. amsthm

                      14. graphics

                      15. aastex

                      16. amscd

                      17. epsf

                      18. color

                      19. aa

                      20. times






                      share|improve this answer



















                      • 28




                        That list is literally pain to my eyes. Loading bm?! Use proper bold math characters instead, please, and not poorman's bold. times? Outdated since ages, use mathptmx or XITS Math instead. I'll stop here...
                        – Ingo
                        Jan 30 '14 at 11:46






                      • 1




                        @Ingo arXiv has been created in 1991 and some papers haven't been updated since then!
                        – Najib Idrissi
                        Feb 24 '17 at 14:57
















                      51














                      For papers on the arXiv (maths, physics and computer science mostly) there's a list of packages sorted by frequency of use.



                      The top twenty packages are:




                      1. article

                      2. graphicx

                      3. amssymb

                      4. amsmath

                      5. revtex

                      6. revtex4

                      7. epsfig

                      8. amsfonts

                      9. bm

                      10. latexsym

                      11. amsart

                      12. dcolumn

                      13. amsthm

                      14. graphics

                      15. aastex

                      16. amscd

                      17. epsf

                      18. color

                      19. aa

                      20. times






                      share|improve this answer



















                      • 28




                        That list is literally pain to my eyes. Loading bm?! Use proper bold math characters instead, please, and not poorman's bold. times? Outdated since ages, use mathptmx or XITS Math instead. I'll stop here...
                        – Ingo
                        Jan 30 '14 at 11:46






                      • 1




                        @Ingo arXiv has been created in 1991 and some papers haven't been updated since then!
                        – Najib Idrissi
                        Feb 24 '17 at 14:57














                      51












                      51








                      51






                      For papers on the arXiv (maths, physics and computer science mostly) there's a list of packages sorted by frequency of use.



                      The top twenty packages are:




                      1. article

                      2. graphicx

                      3. amssymb

                      4. amsmath

                      5. revtex

                      6. revtex4

                      7. epsfig

                      8. amsfonts

                      9. bm

                      10. latexsym

                      11. amsart

                      12. dcolumn

                      13. amsthm

                      14. graphics

                      15. aastex

                      16. amscd

                      17. epsf

                      18. color

                      19. aa

                      20. times






                      share|improve this answer














                      For papers on the arXiv (maths, physics and computer science mostly) there's a list of packages sorted by frequency of use.



                      The top twenty packages are:




                      1. article

                      2. graphicx

                      3. amssymb

                      4. amsmath

                      5. revtex

                      6. revtex4

                      7. epsfig

                      8. amsfonts

                      9. bm

                      10. latexsym

                      11. amsart

                      12. dcolumn

                      13. amsthm

                      14. graphics

                      15. aastex

                      16. amscd

                      17. epsf

                      18. color

                      19. aa

                      20. times







                      share|improve this answer














                      share|improve this answer



                      share|improve this answer








                      edited May 2 '13 at 23:13


























                      community wiki





                      3 revs, 3 users 64%
                      doncherry










                      • 28




                        That list is literally pain to my eyes. Loading bm?! Use proper bold math characters instead, please, and not poorman's bold. times? Outdated since ages, use mathptmx or XITS Math instead. I'll stop here...
                        – Ingo
                        Jan 30 '14 at 11:46






                      • 1




                        @Ingo arXiv has been created in 1991 and some papers haven't been updated since then!
                        – Najib Idrissi
                        Feb 24 '17 at 14:57














                      • 28




                        That list is literally pain to my eyes. Loading bm?! Use proper bold math characters instead, please, and not poorman's bold. times? Outdated since ages, use mathptmx or XITS Math instead. I'll stop here...
                        – Ingo
                        Jan 30 '14 at 11:46






                      • 1




                        @Ingo arXiv has been created in 1991 and some papers haven't been updated since then!
                        – Najib Idrissi
                        Feb 24 '17 at 14:57








                      28




                      28




                      That list is literally pain to my eyes. Loading bm?! Use proper bold math characters instead, please, and not poorman's bold. times? Outdated since ages, use mathptmx or XITS Math instead. I'll stop here...
                      – Ingo
                      Jan 30 '14 at 11:46




                      That list is literally pain to my eyes. Loading bm?! Use proper bold math characters instead, please, and not poorman's bold. times? Outdated since ages, use mathptmx or XITS Math instead. I'll stop here...
                      – Ingo
                      Jan 30 '14 at 11:46




                      1




                      1




                      @Ingo arXiv has been created in 1991 and some papers haven't been updated since then!
                      – Najib Idrissi
                      Feb 24 '17 at 14:57




                      @Ingo arXiv has been created in 1991 and some papers haven't been updated since then!
                      – Najib Idrissi
                      Feb 24 '17 at 14:57











                      44














                      I use url to typeset urls.






                      share|improve this answer




























                        44














                        I use url to typeset urls.






                        share|improve this answer


























                          44












                          44








                          44






                          I use url to typeset urls.






                          share|improve this answer














                          I use url to typeset urls.







                          share|improve this answer














                          share|improve this answer



                          share|improve this answer








                          edited Nov 21 '10 at 22:17


























                          community wiki





                          2 revs, 2 users 67%
                          Seamus

























                              32














                              For quickly setting multicolumn text in a single column document, the multicol package is another package that I use all the time.



                              usepackage{multicol}





                              share|improve this answer




























                                32














                                For quickly setting multicolumn text in a single column document, the multicol package is another package that I use all the time.



                                usepackage{multicol}





                                share|improve this answer


























                                  32












                                  32








                                  32






                                  For quickly setting multicolumn text in a single column document, the multicol package is another package that I use all the time.



                                  usepackage{multicol}





                                  share|improve this answer














                                  For quickly setting multicolumn text in a single column document, the multicol package is another package that I use all the time.



                                  usepackage{multicol}






                                  share|improve this answer














                                  share|improve this answer



                                  share|improve this answer








                                  edited May 5 '11 at 7:43


























                                  community wiki





                                  2 revs, 2 users 80%
                                  Alan Munn

























                                      29














                                      To use the palatino font (it's just a nice looking font)



                                      usepackage[sc]{mathpazo}


                                      Note that the old palatino package is deprecated.






                                      share|improve this answer























                                      • never use usepackage{palatino}, see l2tabu. the current way to use Palatino is usepackage{mathpazo}
                                        – Mateus Araújo
                                        Sep 2 '10 at 3:47










                                      • What is l2tabu?
                                        – Johan
                                        Sep 2 '10 at 6:35






                                      • 13




                                        You should probably also load mathpazo with the [sc] option to get real small caps and better kerning.
                                        – Will Robertson
                                        Sep 2 '10 at 9:24






                                      • 3




                                        Depending on taste, you may want to use [osf] instead of [sc] to get old style numerals as well as the real small caps and better kerning. I for one find old style numerals prettier and classier than lining figures in text mode (using [osf] will keep lining figures in math mode).
                                        – spet
                                        May 29 '13 at 8:52






                                      • 1




                                        According to this the LaTeX font catalogue, one should increase the leading when using mathpazo. tug.dk/FontCatalogue/urwpalladio
                                        – Ubiquitous
                                        Nov 21 '14 at 9:04
















                                      29














                                      To use the palatino font (it's just a nice looking font)



                                      usepackage[sc]{mathpazo}


                                      Note that the old palatino package is deprecated.






                                      share|improve this answer























                                      • never use usepackage{palatino}, see l2tabu. the current way to use Palatino is usepackage{mathpazo}
                                        – Mateus Araújo
                                        Sep 2 '10 at 3:47










                                      • What is l2tabu?
                                        – Johan
                                        Sep 2 '10 at 6:35






                                      • 13




                                        You should probably also load mathpazo with the [sc] option to get real small caps and better kerning.
                                        – Will Robertson
                                        Sep 2 '10 at 9:24






                                      • 3




                                        Depending on taste, you may want to use [osf] instead of [sc] to get old style numerals as well as the real small caps and better kerning. I for one find old style numerals prettier and classier than lining figures in text mode (using [osf] will keep lining figures in math mode).
                                        – spet
                                        May 29 '13 at 8:52






                                      • 1




                                        According to this the LaTeX font catalogue, one should increase the leading when using mathpazo. tug.dk/FontCatalogue/urwpalladio
                                        – Ubiquitous
                                        Nov 21 '14 at 9:04














                                      29












                                      29








                                      29






                                      To use the palatino font (it's just a nice looking font)



                                      usepackage[sc]{mathpazo}


                                      Note that the old palatino package is deprecated.






                                      share|improve this answer














                                      To use the palatino font (it's just a nice looking font)



                                      usepackage[sc]{mathpazo}


                                      Note that the old palatino package is deprecated.







                                      share|improve this answer














                                      share|improve this answer



                                      share|improve this answer








                                      edited Jun 29 '11 at 15:25


























                                      community wiki





                                      4 revs, 3 users 63%
                                      Johan














                                      • never use usepackage{palatino}, see l2tabu. the current way to use Palatino is usepackage{mathpazo}
                                        – Mateus Araújo
                                        Sep 2 '10 at 3:47










                                      • What is l2tabu?
                                        – Johan
                                        Sep 2 '10 at 6:35






                                      • 13




                                        You should probably also load mathpazo with the [sc] option to get real small caps and better kerning.
                                        – Will Robertson
                                        Sep 2 '10 at 9:24






                                      • 3




                                        Depending on taste, you may want to use [osf] instead of [sc] to get old style numerals as well as the real small caps and better kerning. I for one find old style numerals prettier and classier than lining figures in text mode (using [osf] will keep lining figures in math mode).
                                        – spet
                                        May 29 '13 at 8:52






                                      • 1




                                        According to this the LaTeX font catalogue, one should increase the leading when using mathpazo. tug.dk/FontCatalogue/urwpalladio
                                        – Ubiquitous
                                        Nov 21 '14 at 9:04


















                                      • never use usepackage{palatino}, see l2tabu. the current way to use Palatino is usepackage{mathpazo}
                                        – Mateus Araújo
                                        Sep 2 '10 at 3:47










                                      • What is l2tabu?
                                        – Johan
                                        Sep 2 '10 at 6:35






                                      • 13




                                        You should probably also load mathpazo with the [sc] option to get real small caps and better kerning.
                                        – Will Robertson
                                        Sep 2 '10 at 9:24






                                      • 3




                                        Depending on taste, you may want to use [osf] instead of [sc] to get old style numerals as well as the real small caps and better kerning. I for one find old style numerals prettier and classier than lining figures in text mode (using [osf] will keep lining figures in math mode).
                                        – spet
                                        May 29 '13 at 8:52






                                      • 1




                                        According to this the LaTeX font catalogue, one should increase the leading when using mathpazo. tug.dk/FontCatalogue/urwpalladio
                                        – Ubiquitous
                                        Nov 21 '14 at 9:04
















                                      never use usepackage{palatino}, see l2tabu. the current way to use Palatino is usepackage{mathpazo}
                                      – Mateus Araújo
                                      Sep 2 '10 at 3:47




                                      never use usepackage{palatino}, see l2tabu. the current way to use Palatino is usepackage{mathpazo}
                                      – Mateus Araújo
                                      Sep 2 '10 at 3:47












                                      What is l2tabu?
                                      – Johan
                                      Sep 2 '10 at 6:35




                                      What is l2tabu?
                                      – Johan
                                      Sep 2 '10 at 6:35




                                      13




                                      13




                                      You should probably also load mathpazo with the [sc] option to get real small caps and better kerning.
                                      – Will Robertson
                                      Sep 2 '10 at 9:24




                                      You should probably also load mathpazo with the [sc] option to get real small caps and better kerning.
                                      – Will Robertson
                                      Sep 2 '10 at 9:24




                                      3




                                      3




                                      Depending on taste, you may want to use [osf] instead of [sc] to get old style numerals as well as the real small caps and better kerning. I for one find old style numerals prettier and classier than lining figures in text mode (using [osf] will keep lining figures in math mode).
                                      – spet
                                      May 29 '13 at 8:52




                                      Depending on taste, you may want to use [osf] instead of [sc] to get old style numerals as well as the real small caps and better kerning. I for one find old style numerals prettier and classier than lining figures in text mode (using [osf] will keep lining figures in math mode).
                                      – spet
                                      May 29 '13 at 8:52




                                      1




                                      1




                                      According to this the LaTeX font catalogue, one should increase the leading when using mathpazo. tug.dk/FontCatalogue/urwpalladio
                                      – Ubiquitous
                                      Nov 21 '14 at 9:04




                                      According to this the LaTeX font catalogue, one should increase the leading when using mathpazo. tug.dk/FontCatalogue/urwpalladio
                                      – Ubiquitous
                                      Nov 21 '14 at 9:04











                                      29














                                      usepackage[parfill]{parskip} 


                                      I much prefer no indentation and space between paragraphs, so the parskip package is a must for me!






                                      share|improve this answer



















                                      • 17




                                        Have a look at the KOMA-Script-classes - they include a parskip option that is more powerful than the package of the same name.
                                        – lockstep
                                        Aug 8 '10 at 17:39
















                                      29














                                      usepackage[parfill]{parskip} 


                                      I much prefer no indentation and space between paragraphs, so the parskip package is a must for me!






                                      share|improve this answer



















                                      • 17




                                        Have a look at the KOMA-Script-classes - they include a parskip option that is more powerful than the package of the same name.
                                        – lockstep
                                        Aug 8 '10 at 17:39














                                      29












                                      29








                                      29






                                      usepackage[parfill]{parskip} 


                                      I much prefer no indentation and space between paragraphs, so the parskip package is a must for me!






                                      share|improve this answer














                                      usepackage[parfill]{parskip} 


                                      I much prefer no indentation and space between paragraphs, so the parskip package is a must for me!







                                      share|improve this answer














                                      share|improve this answer



                                      share|improve this answer








                                      edited Aug 15 '13 at 19:17


























                                      community wiki





                                      3 revs, 2 users 67%
                                      Vivi










                                      • 17




                                        Have a look at the KOMA-Script-classes - they include a parskip option that is more powerful than the package of the same name.
                                        – lockstep
                                        Aug 8 '10 at 17:39














                                      • 17




                                        Have a look at the KOMA-Script-classes - they include a parskip option that is more powerful than the package of the same name.
                                        – lockstep
                                        Aug 8 '10 at 17:39








                                      17




                                      17




                                      Have a look at the KOMA-Script-classes - they include a parskip option that is more powerful than the package of the same name.
                                      – lockstep
                                      Aug 8 '10 at 17:39




                                      Have a look at the KOMA-Script-classes - they include a parskip option that is more powerful than the package of the same name.
                                      – lockstep
                                      Aug 8 '10 at 17:39











                                      27














                                      I almost always find myself using a tabularx environment as opposed to the regular tabular environment, as it allows for greater dynamism in column widths.






                                      share|improve this answer




























                                        27














                                        I almost always find myself using a tabularx environment as opposed to the regular tabular environment, as it allows for greater dynamism in column widths.






                                        share|improve this answer


























                                          27












                                          27








                                          27






                                          I almost always find myself using a tabularx environment as opposed to the regular tabular environment, as it allows for greater dynamism in column widths.






                                          share|improve this answer














                                          I almost always find myself using a tabularx environment as opposed to the regular tabular environment, as it allows for greater dynamism in column widths.







                                          share|improve this answer














                                          share|improve this answer



                                          share|improve this answer








                                          answered Dec 29 '10 at 0:49


























                                          community wiki





                                          ESultanik
























                                              27














                                              Nothing surprising here: I use natbib, hyperref and hypernat together.



                                              Natbib for referencing.



                                              Hyperref adds bookmarks for sections and lists and turns references and urls into links.



                                              Hypernat allows natbib and hyperref to work together. -- Note (added 2015/02/11): natbib and hyperref have been working together just fine for at least ten years. hypernat is no longer needed for any TeX distribution with a vintage more recent than ca 2002.






                                              share|improve this answer



















                                              • 8




                                                I'm pretty sure that hypernat is superfluous these days. With only loading natbib and hyperref I get references as [1-5] with both 1 and 5 being hyperlinks.
                                                – Lev Bishop
                                                Aug 8 '10 at 14:51










                                              • Agreed, I didn't even know about hypernat until I saw this answer. I have been using hyperref and natbib for a while and reference links and backlinks always worked for me. Is there some extra functionality that hypernat adds?
                                                – Sharpie
                                                Aug 9 '10 at 17:31










                                              • I had a problem once, found out about natbib, and have been using it ever since, so it is possible it is superfluous and I didn't even know. I will have to test it out and get back to you guys if I find something.
                                                – Vivi
                                                Aug 10 '10 at 20:18






                                              • 2




                                                And? Was it superfluous in 2010? Is it now? ;)
                                                – K.-Michael Aye
                                                Nov 23 '12 at 5:18






                                              • 1




                                                @K.-MichaelAye - hypernat was superfluous (and potentially troublesome) back in 2010 and in 2012, and it continues to be superfluous as of 2015.
                                                – Mico
                                                Feb 11 '15 at 21:13
















                                              27














                                              Nothing surprising here: I use natbib, hyperref and hypernat together.



                                              Natbib for referencing.



                                              Hyperref adds bookmarks for sections and lists and turns references and urls into links.



                                              Hypernat allows natbib and hyperref to work together. -- Note (added 2015/02/11): natbib and hyperref have been working together just fine for at least ten years. hypernat is no longer needed for any TeX distribution with a vintage more recent than ca 2002.






                                              share|improve this answer



















                                              • 8




                                                I'm pretty sure that hypernat is superfluous these days. With only loading natbib and hyperref I get references as [1-5] with both 1 and 5 being hyperlinks.
                                                – Lev Bishop
                                                Aug 8 '10 at 14:51










                                              • Agreed, I didn't even know about hypernat until I saw this answer. I have been using hyperref and natbib for a while and reference links and backlinks always worked for me. Is there some extra functionality that hypernat adds?
                                                – Sharpie
                                                Aug 9 '10 at 17:31










                                              • I had a problem once, found out about natbib, and have been using it ever since, so it is possible it is superfluous and I didn't even know. I will have to test it out and get back to you guys if I find something.
                                                – Vivi
                                                Aug 10 '10 at 20:18






                                              • 2




                                                And? Was it superfluous in 2010? Is it now? ;)
                                                – K.-Michael Aye
                                                Nov 23 '12 at 5:18






                                              • 1




                                                @K.-MichaelAye - hypernat was superfluous (and potentially troublesome) back in 2010 and in 2012, and it continues to be superfluous as of 2015.
                                                – Mico
                                                Feb 11 '15 at 21:13














                                              27












                                              27








                                              27






                                              Nothing surprising here: I use natbib, hyperref and hypernat together.



                                              Natbib for referencing.



                                              Hyperref adds bookmarks for sections and lists and turns references and urls into links.



                                              Hypernat allows natbib and hyperref to work together. -- Note (added 2015/02/11): natbib and hyperref have been working together just fine for at least ten years. hypernat is no longer needed for any TeX distribution with a vintage more recent than ca 2002.






                                              share|improve this answer














                                              Nothing surprising here: I use natbib, hyperref and hypernat together.



                                              Natbib for referencing.



                                              Hyperref adds bookmarks for sections and lists and turns references and urls into links.



                                              Hypernat allows natbib and hyperref to work together. -- Note (added 2015/02/11): natbib and hyperref have been working together just fine for at least ten years. hypernat is no longer needed for any TeX distribution with a vintage more recent than ca 2002.







                                              share|improve this answer














                                              share|improve this answer



                                              share|improve this answer








                                              edited Feb 11 '15 at 20:01


























                                              community wiki





                                              4 revs, 3 users 88%
                                              Vivi









                                              • 8




                                                I'm pretty sure that hypernat is superfluous these days. With only loading natbib and hyperref I get references as [1-5] with both 1 and 5 being hyperlinks.
                                                – Lev Bishop
                                                Aug 8 '10 at 14:51










                                              • Agreed, I didn't even know about hypernat until I saw this answer. I have been using hyperref and natbib for a while and reference links and backlinks always worked for me. Is there some extra functionality that hypernat adds?
                                                – Sharpie
                                                Aug 9 '10 at 17:31










                                              • I had a problem once, found out about natbib, and have been using it ever since, so it is possible it is superfluous and I didn't even know. I will have to test it out and get back to you guys if I find something.
                                                – Vivi
                                                Aug 10 '10 at 20:18






                                              • 2




                                                And? Was it superfluous in 2010? Is it now? ;)
                                                – K.-Michael Aye
                                                Nov 23 '12 at 5:18






                                              • 1




                                                @K.-MichaelAye - hypernat was superfluous (and potentially troublesome) back in 2010 and in 2012, and it continues to be superfluous as of 2015.
                                                – Mico
                                                Feb 11 '15 at 21:13














                                              • 8




                                                I'm pretty sure that hypernat is superfluous these days. With only loading natbib and hyperref I get references as [1-5] with both 1 and 5 being hyperlinks.
                                                – Lev Bishop
                                                Aug 8 '10 at 14:51










                                              • Agreed, I didn't even know about hypernat until I saw this answer. I have been using hyperref and natbib for a while and reference links and backlinks always worked for me. Is there some extra functionality that hypernat adds?
                                                – Sharpie
                                                Aug 9 '10 at 17:31










                                              • I had a problem once, found out about natbib, and have been using it ever since, so it is possible it is superfluous and I didn't even know. I will have to test it out and get back to you guys if I find something.
                                                – Vivi
                                                Aug 10 '10 at 20:18






                                              • 2




                                                And? Was it superfluous in 2010? Is it now? ;)
                                                – K.-Michael Aye
                                                Nov 23 '12 at 5:18






                                              • 1




                                                @K.-MichaelAye - hypernat was superfluous (and potentially troublesome) back in 2010 and in 2012, and it continues to be superfluous as of 2015.
                                                – Mico
                                                Feb 11 '15 at 21:13








                                              8




                                              8




                                              I'm pretty sure that hypernat is superfluous these days. With only loading natbib and hyperref I get references as [1-5] with both 1 and 5 being hyperlinks.
                                              – Lev Bishop
                                              Aug 8 '10 at 14:51




                                              I'm pretty sure that hypernat is superfluous these days. With only loading natbib and hyperref I get references as [1-5] with both 1 and 5 being hyperlinks.
                                              – Lev Bishop
                                              Aug 8 '10 at 14:51












                                              Agreed, I didn't even know about hypernat until I saw this answer. I have been using hyperref and natbib for a while and reference links and backlinks always worked for me. Is there some extra functionality that hypernat adds?
                                              – Sharpie
                                              Aug 9 '10 at 17:31




                                              Agreed, I didn't even know about hypernat until I saw this answer. I have been using hyperref and natbib for a while and reference links and backlinks always worked for me. Is there some extra functionality that hypernat adds?
                                              – Sharpie
                                              Aug 9 '10 at 17:31












                                              I had a problem once, found out about natbib, and have been using it ever since, so it is possible it is superfluous and I didn't even know. I will have to test it out and get back to you guys if I find something.
                                              – Vivi
                                              Aug 10 '10 at 20:18




                                              I had a problem once, found out about natbib, and have been using it ever since, so it is possible it is superfluous and I didn't even know. I will have to test it out and get back to you guys if I find something.
                                              – Vivi
                                              Aug 10 '10 at 20:18




                                              2




                                              2




                                              And? Was it superfluous in 2010? Is it now? ;)
                                              – K.-Michael Aye
                                              Nov 23 '12 at 5:18




                                              And? Was it superfluous in 2010? Is it now? ;)
                                              – K.-Michael Aye
                                              Nov 23 '12 at 5:18




                                              1




                                              1




                                              @K.-MichaelAye - hypernat was superfluous (and potentially troublesome) back in 2010 and in 2012, and it continues to be superfluous as of 2015.
                                              – Mico
                                              Feb 11 '15 at 21:13




                                              @K.-MichaelAye - hypernat was superfluous (and potentially troublesome) back in 2010 and in 2012, and it continues to be superfluous as of 2015.
                                              – Mico
                                              Feb 11 '15 at 21:13











                                              25














                                              I almost always use the enumitem package, which makes it much easier to make modifications to lists (especially enumerate lists). Most notably, changing the labels to something like (i), (ii), (iii) [no period] with this package is as easy as



                                              begin{enumerate}[label=(roman*)]
                                              item The first item
                                              item The second item
                                              end{enumerate}


                                              Furthermore, the code above will automatically get nesting right. Before I started using this package, my preamble always included the awkward macro (necessary to change the references and eliminate the extra period in the list itself)



                                              newcommand{setenumroman}{%
                                              renewcommand{theenumi}{(roman{enumi})}%
                                              renewcommand{labelenumi}{theenumi}%
                                              }


                                              which would break if I ever used it for a nested list (all the enumis would have to be changed to enumiis, if I understand correctly).



                                              The enumitem package is quite flexible; another option I sometimes use is [wide], which makes a list look like part of the body of the text (with numbers/labels at the beginning of relevant paragraphs).






                                              share|improve this answer























                                              • If someone only want the feature of changing labels, easier will be to use the enumerate package. Then you could simply write begin{enumerate}[(i)]. But enumitem package gives a lot more flexibility including allowing the items to appear in a line.
                                                – Cyriac Antony
                                                Dec 21 '18 at 8:38


















                                              25














                                              I almost always use the enumitem package, which makes it much easier to make modifications to lists (especially enumerate lists). Most notably, changing the labels to something like (i), (ii), (iii) [no period] with this package is as easy as



                                              begin{enumerate}[label=(roman*)]
                                              item The first item
                                              item The second item
                                              end{enumerate}


                                              Furthermore, the code above will automatically get nesting right. Before I started using this package, my preamble always included the awkward macro (necessary to change the references and eliminate the extra period in the list itself)



                                              newcommand{setenumroman}{%
                                              renewcommand{theenumi}{(roman{enumi})}%
                                              renewcommand{labelenumi}{theenumi}%
                                              }


                                              which would break if I ever used it for a nested list (all the enumis would have to be changed to enumiis, if I understand correctly).



                                              The enumitem package is quite flexible; another option I sometimes use is [wide], which makes a list look like part of the body of the text (with numbers/labels at the beginning of relevant paragraphs).






                                              share|improve this answer























                                              • If someone only want the feature of changing labels, easier will be to use the enumerate package. Then you could simply write begin{enumerate}[(i)]. But enumitem package gives a lot more flexibility including allowing the items to appear in a line.
                                                – Cyriac Antony
                                                Dec 21 '18 at 8:38
















                                              25












                                              25








                                              25






                                              I almost always use the enumitem package, which makes it much easier to make modifications to lists (especially enumerate lists). Most notably, changing the labels to something like (i), (ii), (iii) [no period] with this package is as easy as



                                              begin{enumerate}[label=(roman*)]
                                              item The first item
                                              item The second item
                                              end{enumerate}


                                              Furthermore, the code above will automatically get nesting right. Before I started using this package, my preamble always included the awkward macro (necessary to change the references and eliminate the extra period in the list itself)



                                              newcommand{setenumroman}{%
                                              renewcommand{theenumi}{(roman{enumi})}%
                                              renewcommand{labelenumi}{theenumi}%
                                              }


                                              which would break if I ever used it for a nested list (all the enumis would have to be changed to enumiis, if I understand correctly).



                                              The enumitem package is quite flexible; another option I sometimes use is [wide], which makes a list look like part of the body of the text (with numbers/labels at the beginning of relevant paragraphs).






                                              share|improve this answer














                                              I almost always use the enumitem package, which makes it much easier to make modifications to lists (especially enumerate lists). Most notably, changing the labels to something like (i), (ii), (iii) [no period] with this package is as easy as



                                              begin{enumerate}[label=(roman*)]
                                              item The first item
                                              item The second item
                                              end{enumerate}


                                              Furthermore, the code above will automatically get nesting right. Before I started using this package, my preamble always included the awkward macro (necessary to change the references and eliminate the extra period in the list itself)



                                              newcommand{setenumroman}{%
                                              renewcommand{theenumi}{(roman{enumi})}%
                                              renewcommand{labelenumi}{theenumi}%
                                              }


                                              which would break if I ever used it for a nested list (all the enumis would have to be changed to enumiis, if I understand correctly).



                                              The enumitem package is quite flexible; another option I sometimes use is [wide], which makes a list look like part of the body of the text (with numbers/labels at the beginning of relevant paragraphs).







                                              share|improve this answer














                                              share|improve this answer



                                              share|improve this answer








                                              answered Dec 6 '12 at 3:19


























                                              community wiki





                                              Charles Staats













                                              • If someone only want the feature of changing labels, easier will be to use the enumerate package. Then you could simply write begin{enumerate}[(i)]. But enumitem package gives a lot more flexibility including allowing the items to appear in a line.
                                                – Cyriac Antony
                                                Dec 21 '18 at 8:38




















                                              • If someone only want the feature of changing labels, easier will be to use the enumerate package. Then you could simply write begin{enumerate}[(i)]. But enumitem package gives a lot more flexibility including allowing the items to appear in a line.
                                                – Cyriac Antony
                                                Dec 21 '18 at 8:38


















                                              If someone only want the feature of changing labels, easier will be to use the enumerate package. Then you could simply write begin{enumerate}[(i)]. But enumitem package gives a lot more flexibility including allowing the items to appear in a line.
                                              – Cyriac Antony
                                              Dec 21 '18 at 8:38






                                              If someone only want the feature of changing labels, easier will be to use the enumerate package. Then you could simply write begin{enumerate}[(i)]. But enumitem package gives a lot more flexibility including allowing the items to appear in a line.
                                              – Cyriac Antony
                                              Dec 21 '18 at 8:38













                                              24














                                              To make sure you have ISO formated dates (YYYY-MM-DD).



                                              usepackage[english]{isodate}


                                              or



                                              usepackage{datetime}
                                              renewcommand{dateseparator}{-}
                                              newcommand{todayiso}{theyear dateseparator twodigitmonth dateseparator twodigitday}





                                              share|improve this answer




























                                                24














                                                To make sure you have ISO formated dates (YYYY-MM-DD).



                                                usepackage[english]{isodate}


                                                or



                                                usepackage{datetime}
                                                renewcommand{dateseparator}{-}
                                                newcommand{todayiso}{theyear dateseparator twodigitmonth dateseparator twodigitday}





                                                share|improve this answer


























                                                  24












                                                  24








                                                  24






                                                  To make sure you have ISO formated dates (YYYY-MM-DD).



                                                  usepackage[english]{isodate}


                                                  or



                                                  usepackage{datetime}
                                                  renewcommand{dateseparator}{-}
                                                  newcommand{todayiso}{theyear dateseparator twodigitmonth dateseparator twodigitday}





                                                  share|improve this answer














                                                  To make sure you have ISO formated dates (YYYY-MM-DD).



                                                  usepackage[english]{isodate}


                                                  or



                                                  usepackage{datetime}
                                                  renewcommand{dateseparator}{-}
                                                  newcommand{todayiso}{theyear dateseparator twodigitmonth dateseparator twodigitday}






                                                  share|improve this answer














                                                  share|improve this answer



                                                  share|improve this answer








                                                  answered Aug 15 '10 at 9:57


























                                                  community wiki





                                                  Johan
























                                                      20














                                                      Another package I use is float. It allows for the placement H for floats, which is somewhat equivalent to h!, but a bit stronger, making sure the figure or table goes exactly where I want it to be.






                                                      share|improve this answer



















                                                      • 6




                                                        Actually not equivalent to h! at all. h! floats still "float"- they can be moved around by LaTeX in an attempt to optimize the document layout. Figures using the H specifier are not floats at all, they are treated like one big character and are put exactly where they appear in the text.
                                                        – Sharpie
                                                        Aug 1 '10 at 3:59












                                                      • @Sharpie: you are ignoring the word "somewhat" :P Still, your point is valid, thanks!
                                                        – Vivi
                                                        Aug 1 '10 at 4:21






                                                      • 1




                                                        I did consider the word somewhat. However, in my opinion the only similarity between the two is the fact that they are used as float specifiers. Beyond that, both specifiers produce completely different effects.
                                                        – Sharpie
                                                        Aug 1 '10 at 6:14










                                                      • @Sharpie: maybe I should link to the source of the (mis)information? en.wikibooks.org/wiki/LaTeX/… (see the last row of the table)
                                                        – Vivi
                                                        Aug 1 '10 at 6:32










                                                      • @Vivi I fixed that entry of the wikibook.
                                                        – Skillmon
                                                        Dec 15 '18 at 10:46
















                                                      20














                                                      Another package I use is float. It allows for the placement H for floats, which is somewhat equivalent to h!, but a bit stronger, making sure the figure or table goes exactly where I want it to be.






                                                      share|improve this answer



















                                                      • 6




                                                        Actually not equivalent to h! at all. h! floats still "float"- they can be moved around by LaTeX in an attempt to optimize the document layout. Figures using the H specifier are not floats at all, they are treated like one big character and are put exactly where they appear in the text.
                                                        – Sharpie
                                                        Aug 1 '10 at 3:59












                                                      • @Sharpie: you are ignoring the word "somewhat" :P Still, your point is valid, thanks!
                                                        – Vivi
                                                        Aug 1 '10 at 4:21






                                                      • 1




                                                        I did consider the word somewhat. However, in my opinion the only similarity between the two is the fact that they are used as float specifiers. Beyond that, both specifiers produce completely different effects.
                                                        – Sharpie
                                                        Aug 1 '10 at 6:14










                                                      • @Sharpie: maybe I should link to the source of the (mis)information? en.wikibooks.org/wiki/LaTeX/… (see the last row of the table)
                                                        – Vivi
                                                        Aug 1 '10 at 6:32










                                                      • @Vivi I fixed that entry of the wikibook.
                                                        – Skillmon
                                                        Dec 15 '18 at 10:46














                                                      20












                                                      20








                                                      20






                                                      Another package I use is float. It allows for the placement H for floats, which is somewhat equivalent to h!, but a bit stronger, making sure the figure or table goes exactly where I want it to be.






                                                      share|improve this answer














                                                      Another package I use is float. It allows for the placement H for floats, which is somewhat equivalent to h!, but a bit stronger, making sure the figure or table goes exactly where I want it to be.







                                                      share|improve this answer














                                                      share|improve this answer



                                                      share|improve this answer








                                                      edited Feb 7 '13 at 4:21


























                                                      community wiki





                                                      3 revs, 3 users 57%
                                                      doncherry










                                                      • 6




                                                        Actually not equivalent to h! at all. h! floats still "float"- they can be moved around by LaTeX in an attempt to optimize the document layout. Figures using the H specifier are not floats at all, they are treated like one big character and are put exactly where they appear in the text.
                                                        – Sharpie
                                                        Aug 1 '10 at 3:59












                                                      • @Sharpie: you are ignoring the word "somewhat" :P Still, your point is valid, thanks!
                                                        – Vivi
                                                        Aug 1 '10 at 4:21






                                                      • 1




                                                        I did consider the word somewhat. However, in my opinion the only similarity between the two is the fact that they are used as float specifiers. Beyond that, both specifiers produce completely different effects.
                                                        – Sharpie
                                                        Aug 1 '10 at 6:14










                                                      • @Sharpie: maybe I should link to the source of the (mis)information? en.wikibooks.org/wiki/LaTeX/… (see the last row of the table)
                                                        – Vivi
                                                        Aug 1 '10 at 6:32










                                                      • @Vivi I fixed that entry of the wikibook.
                                                        – Skillmon
                                                        Dec 15 '18 at 10:46














                                                      • 6




                                                        Actually not equivalent to h! at all. h! floats still "float"- they can be moved around by LaTeX in an attempt to optimize the document layout. Figures using the H specifier are not floats at all, they are treated like one big character and are put exactly where they appear in the text.
                                                        – Sharpie
                                                        Aug 1 '10 at 3:59












                                                      • @Sharpie: you are ignoring the word "somewhat" :P Still, your point is valid, thanks!
                                                        – Vivi
                                                        Aug 1 '10 at 4:21






                                                      • 1




                                                        I did consider the word somewhat. However, in my opinion the only similarity between the two is the fact that they are used as float specifiers. Beyond that, both specifiers produce completely different effects.
                                                        – Sharpie
                                                        Aug 1 '10 at 6:14










                                                      • @Sharpie: maybe I should link to the source of the (mis)information? en.wikibooks.org/wiki/LaTeX/… (see the last row of the table)
                                                        – Vivi
                                                        Aug 1 '10 at 6:32










                                                      • @Vivi I fixed that entry of the wikibook.
                                                        – Skillmon
                                                        Dec 15 '18 at 10:46








                                                      6




                                                      6




                                                      Actually not equivalent to h! at all. h! floats still "float"- they can be moved around by LaTeX in an attempt to optimize the document layout. Figures using the H specifier are not floats at all, they are treated like one big character and are put exactly where they appear in the text.
                                                      – Sharpie
                                                      Aug 1 '10 at 3:59






                                                      Actually not equivalent to h! at all. h! floats still "float"- they can be moved around by LaTeX in an attempt to optimize the document layout. Figures using the H specifier are not floats at all, they are treated like one big character and are put exactly where they appear in the text.
                                                      – Sharpie
                                                      Aug 1 '10 at 3:59














                                                      @Sharpie: you are ignoring the word "somewhat" :P Still, your point is valid, thanks!
                                                      – Vivi
                                                      Aug 1 '10 at 4:21




                                                      @Sharpie: you are ignoring the word "somewhat" :P Still, your point is valid, thanks!
                                                      – Vivi
                                                      Aug 1 '10 at 4:21




                                                      1




                                                      1




                                                      I did consider the word somewhat. However, in my opinion the only similarity between the two is the fact that they are used as float specifiers. Beyond that, both specifiers produce completely different effects.
                                                      – Sharpie
                                                      Aug 1 '10 at 6:14




                                                      I did consider the word somewhat. However, in my opinion the only similarity between the two is the fact that they are used as float specifiers. Beyond that, both specifiers produce completely different effects.
                                                      – Sharpie
                                                      Aug 1 '10 at 6:14












                                                      @Sharpie: maybe I should link to the source of the (mis)information? en.wikibooks.org/wiki/LaTeX/… (see the last row of the table)
                                                      – Vivi
                                                      Aug 1 '10 at 6:32




                                                      @Sharpie: maybe I should link to the source of the (mis)information? en.wikibooks.org/wiki/LaTeX/… (see the last row of the table)
                                                      – Vivi
                                                      Aug 1 '10 at 6:32












                                                      @Vivi I fixed that entry of the wikibook.
                                                      – Skillmon
                                                      Dec 15 '18 at 10:46




                                                      @Vivi I fixed that entry of the wikibook.
                                                      – Skillmon
                                                      Dec 15 '18 at 10:46











                                                      19














                                                      For mathematical texts I instead use amsmath & Co. One very useful package is onlyamsmath. I load it as



                                                      usepackage[all,warning]{onlyamsmath}


                                                      So it looks for $$..$$, eqnarray and produces a warning if some of them are used. If you left out warning, it will result in an error and compile will stop. This package is normally very useful if you edit a text with many authors.






                                                      share|improve this answer




























                                                        19














                                                        For mathematical texts I instead use amsmath & Co. One very useful package is onlyamsmath. I load it as



                                                        usepackage[all,warning]{onlyamsmath}


                                                        So it looks for $$..$$, eqnarray and produces a warning if some of them are used. If you left out warning, it will result in an error and compile will stop. This package is normally very useful if you edit a text with many authors.






                                                        share|improve this answer


























                                                          19












                                                          19








                                                          19






                                                          For mathematical texts I instead use amsmath & Co. One very useful package is onlyamsmath. I load it as



                                                          usepackage[all,warning]{onlyamsmath}


                                                          So it looks for $$..$$, eqnarray and produces a warning if some of them are used. If you left out warning, it will result in an error and compile will stop. This package is normally very useful if you edit a text with many authors.






                                                          share|improve this answer














                                                          For mathematical texts I instead use amsmath & Co. One very useful package is onlyamsmath. I load it as



                                                          usepackage[all,warning]{onlyamsmath}


                                                          So it looks for $$..$$, eqnarray and produces a warning if some of them are used. If you left out warning, it will result in an error and compile will stop. This package is normally very useful if you edit a text with many authors.







                                                          share|improve this answer














                                                          share|improve this answer



                                                          share|improve this answer








                                                          answered Jul 30 '10 at 19:05


























                                                          community wiki





                                                          qbi
























                                                              19














                                                              Edited by doncherry: Removed packages mentioned in separate answers.



                                                              The complete header Part of my header for most of my documents looks as follows:



                                                              documentclass[ngerman,draft,parskip=half*,twoside]{scrreprt}
                                                              usepackage{ifthen}


                                                              For some things I need if-then-constructs. This package provides an easy way to realise it.



                                                              usepackage{index}


                                                              For generating an index.



                                                              usepackage{xcolor}


                                                              xcolor is needed by several packages. For some historical reason I load it manually.



                                                              usepackage{babel}
                                                              usepackage{nicefrac}


                                                              nicefrac allows typesetting fractions like 1/2. It is sometimes more readable than frac.



                                                              usepackage[T1]{fontenc}
                                                              usepackage[intlimits,leqno]{amsmath}
                                                              usepackage[all,warning]{onlyamsmath}


                                                              This package warns if non-amsmath-environments are used.



                                                              usepackage{amssymb}
                                                              usepackage{fixmath}


                                                              Provides ISO conform greek letters.



                                                              usepackage[euro]{isonums}


                                                              Defines comma as decimal delimiter.



                                                              usepackage[amsmath,thmmarks,hyperref]{ntheorem}


                                                              for Theorems, definitions and stuff.



                                                              usepackage{paralist}


                                                              Improves enumerate and itemize. Also provides some compact environments.



                                                              usepackage{svn}


                                                              I work with VCS and svn displays some informations (keywords) from SVN.



                                                              usepackage{ellipsis}


                                                              corrects dots



                                                              DeclarePairedDelimiter{abs}{lvert}{rvert}
                                                              DeclarePairedDelimiter{norm}{lVert}{rVert}


                                                              These are the definitions for absolute value and norm.



                                                              SVN $LastChangedRevision$
                                                              SVN $LastChangedDate$





                                                              share|improve this answer



















                                                              • 26




                                                                "one thing (in this case, package) per answer"
                                                                – Jukka Suomela
                                                                Jul 29 '10 at 19:02






                                                              • 4




                                                                Could you break this up into multiple answers please, so they can be voted on? Having a dozen answers is ok!
                                                                – ShreevatsaR
                                                                Jul 30 '10 at 14:41










                                                              • Excellent list with helpful commentary. I will add the one for old and deprecated things to my list. Thanks!
                                                                – DJP
                                                                Jul 30 '11 at 20:31






                                                              • 3




                                                                It is usually recommended to load hyperref last.
                                                                – Alex Hirzel
                                                                May 1 '12 at 20:20
















                                                              19














                                                              Edited by doncherry: Removed packages mentioned in separate answers.



                                                              The complete header Part of my header for most of my documents looks as follows:



                                                              documentclass[ngerman,draft,parskip=half*,twoside]{scrreprt}
                                                              usepackage{ifthen}


                                                              For some things I need if-then-constructs. This package provides an easy way to realise it.



                                                              usepackage{index}


                                                              For generating an index.



                                                              usepackage{xcolor}


                                                              xcolor is needed by several packages. For some historical reason I load it manually.



                                                              usepackage{babel}
                                                              usepackage{nicefrac}


                                                              nicefrac allows typesetting fractions like 1/2. It is sometimes more readable than frac.



                                                              usepackage[T1]{fontenc}
                                                              usepackage[intlimits,leqno]{amsmath}
                                                              usepackage[all,warning]{onlyamsmath}


                                                              This package warns if non-amsmath-environments are used.



                                                              usepackage{amssymb}
                                                              usepackage{fixmath}


                                                              Provides ISO conform greek letters.



                                                              usepackage[euro]{isonums}


                                                              Defines comma as decimal delimiter.



                                                              usepackage[amsmath,thmmarks,hyperref]{ntheorem}


                                                              for Theorems, definitions and stuff.



                                                              usepackage{paralist}


                                                              Improves enumerate and itemize. Also provides some compact environments.



                                                              usepackage{svn}


                                                              I work with VCS and svn displays some informations (keywords) from SVN.



                                                              usepackage{ellipsis}


                                                              corrects dots



                                                              DeclarePairedDelimiter{abs}{lvert}{rvert}
                                                              DeclarePairedDelimiter{norm}{lVert}{rVert}


                                                              These are the definitions for absolute value and norm.



                                                              SVN $LastChangedRevision$
                                                              SVN $LastChangedDate$





                                                              share|improve this answer



















                                                              • 26




                                                                "one thing (in this case, package) per answer"
                                                                – Jukka Suomela
                                                                Jul 29 '10 at 19:02






                                                              • 4




                                                                Could you break this up into multiple answers please, so they can be voted on? Having a dozen answers is ok!
                                                                – ShreevatsaR
                                                                Jul 30 '10 at 14:41










                                                              • Excellent list with helpful commentary. I will add the one for old and deprecated things to my list. Thanks!
                                                                – DJP
                                                                Jul 30 '11 at 20:31






                                                              • 3




                                                                It is usually recommended to load hyperref last.
                                                                – Alex Hirzel
                                                                May 1 '12 at 20:20














                                                              19












                                                              19








                                                              19






                                                              Edited by doncherry: Removed packages mentioned in separate answers.



                                                              The complete header Part of my header for most of my documents looks as follows:



                                                              documentclass[ngerman,draft,parskip=half*,twoside]{scrreprt}
                                                              usepackage{ifthen}


                                                              For some things I need if-then-constructs. This package provides an easy way to realise it.



                                                              usepackage{index}


                                                              For generating an index.



                                                              usepackage{xcolor}


                                                              xcolor is needed by several packages. For some historical reason I load it manually.



                                                              usepackage{babel}
                                                              usepackage{nicefrac}


                                                              nicefrac allows typesetting fractions like 1/2. It is sometimes more readable than frac.



                                                              usepackage[T1]{fontenc}
                                                              usepackage[intlimits,leqno]{amsmath}
                                                              usepackage[all,warning]{onlyamsmath}


                                                              This package warns if non-amsmath-environments are used.



                                                              usepackage{amssymb}
                                                              usepackage{fixmath}


                                                              Provides ISO conform greek letters.



                                                              usepackage[euro]{isonums}


                                                              Defines comma as decimal delimiter.



                                                              usepackage[amsmath,thmmarks,hyperref]{ntheorem}


                                                              for Theorems, definitions and stuff.



                                                              usepackage{paralist}


                                                              Improves enumerate and itemize. Also provides some compact environments.



                                                              usepackage{svn}


                                                              I work with VCS and svn displays some informations (keywords) from SVN.



                                                              usepackage{ellipsis}


                                                              corrects dots



                                                              DeclarePairedDelimiter{abs}{lvert}{rvert}
                                                              DeclarePairedDelimiter{norm}{lVert}{rVert}


                                                              These are the definitions for absolute value and norm.



                                                              SVN $LastChangedRevision$
                                                              SVN $LastChangedDate$





                                                              share|improve this answer














                                                              Edited by doncherry: Removed packages mentioned in separate answers.



                                                              The complete header Part of my header for most of my documents looks as follows:



                                                              documentclass[ngerman,draft,parskip=half*,twoside]{scrreprt}
                                                              usepackage{ifthen}


                                                              For some things I need if-then-constructs. This package provides an easy way to realise it.



                                                              usepackage{index}


                                                              For generating an index.



                                                              usepackage{xcolor}


                                                              xcolor is needed by several packages. For some historical reason I load it manually.



                                                              usepackage{babel}
                                                              usepackage{nicefrac}


                                                              nicefrac allows typesetting fractions like 1/2. It is sometimes more readable than frac.



                                                              usepackage[T1]{fontenc}
                                                              usepackage[intlimits,leqno]{amsmath}
                                                              usepackage[all,warning]{onlyamsmath}


                                                              This package warns if non-amsmath-environments are used.



                                                              usepackage{amssymb}
                                                              usepackage{fixmath}


                                                              Provides ISO conform greek letters.



                                                              usepackage[euro]{isonums}


                                                              Defines comma as decimal delimiter.



                                                              usepackage[amsmath,thmmarks,hyperref]{ntheorem}


                                                              for Theorems, definitions and stuff.



                                                              usepackage{paralist}


                                                              Improves enumerate and itemize. Also provides some compact environments.



                                                              usepackage{svn}


                                                              I work with VCS and svn displays some informations (keywords) from SVN.



                                                              usepackage{ellipsis}


                                                              corrects dots



                                                              DeclarePairedDelimiter{abs}{lvert}{rvert}
                                                              DeclarePairedDelimiter{norm}{lVert}{rVert}


                                                              These are the definitions for absolute value and norm.



                                                              SVN $LastChangedRevision$
                                                              SVN $LastChangedDate$






                                                              share|improve this answer














                                                              share|improve this answer



                                                              share|improve this answer








                                                              edited Jul 21 '12 at 11:36


























                                                              community wiki





                                                              3 revs, 3 users 85%
                                                              qbi










                                                              • 26




                                                                "one thing (in this case, package) per answer"
                                                                – Jukka Suomela
                                                                Jul 29 '10 at 19:02






                                                              • 4




                                                                Could you break this up into multiple answers please, so they can be voted on? Having a dozen answers is ok!
                                                                – ShreevatsaR
                                                                Jul 30 '10 at 14:41










                                                              • Excellent list with helpful commentary. I will add the one for old and deprecated things to my list. Thanks!
                                                                – DJP
                                                                Jul 30 '11 at 20:31






                                                              • 3




                                                                It is usually recommended to load hyperref last.
                                                                – Alex Hirzel
                                                                May 1 '12 at 20:20














                                                              • 26




                                                                "one thing (in this case, package) per answer"
                                                                – Jukka Suomela
                                                                Jul 29 '10 at 19:02






                                                              • 4




                                                                Could you break this up into multiple answers please, so they can be voted on? Having a dozen answers is ok!
                                                                – ShreevatsaR
                                                                Jul 30 '10 at 14:41










                                                              • Excellent list with helpful commentary. I will add the one for old and deprecated things to my list. Thanks!
                                                                – DJP
                                                                Jul 30 '11 at 20:31






                                                              • 3




                                                                It is usually recommended to load hyperref last.
                                                                – Alex Hirzel
                                                                May 1 '12 at 20:20








                                                              26




                                                              26




                                                              "one thing (in this case, package) per answer"
                                                              – Jukka Suomela
                                                              Jul 29 '10 at 19:02




                                                              "one thing (in this case, package) per answer"
                                                              – Jukka Suomela
                                                              Jul 29 '10 at 19:02




                                                              4




                                                              4




                                                              Could you break this up into multiple answers please, so they can be voted on? Having a dozen answers is ok!
                                                              – ShreevatsaR
                                                              Jul 30 '10 at 14:41




                                                              Could you break this up into multiple answers please, so they can be voted on? Having a dozen answers is ok!
                                                              – ShreevatsaR
                                                              Jul 30 '10 at 14:41












                                                              Excellent list with helpful commentary. I will add the one for old and deprecated things to my list. Thanks!
                                                              – DJP
                                                              Jul 30 '11 at 20:31




                                                              Excellent list with helpful commentary. I will add the one for old and deprecated things to my list. Thanks!
                                                              – DJP
                                                              Jul 30 '11 at 20:31




                                                              3




                                                              3




                                                              It is usually recommended to load hyperref last.
                                                              – Alex Hirzel
                                                              May 1 '12 at 20:20




                                                              It is usually recommended to load hyperref last.
                                                              – Alex Hirzel
                                                              May 1 '12 at 20:20










                                                              1 2
                                                              3
                                                              next

















                                                              draft saved

                                                              draft discarded




















































                                                              Thanks for contributing an answer to TeX - LaTeX Stack Exchange!


                                                              • Please be sure to answer the question. Provide details and share your research!

                                                              But avoid



                                                              • Asking for help, clarification, or responding to other answers.

                                                              • Making statements based on opinion; back them up with references or personal experience.


                                                              To learn more, see our tips on writing great answers.





                                                              Some of your past answers have not been well-received, and you're in danger of being blocked from answering.


                                                              Please pay close attention to the following guidance:


                                                              • Please be sure to answer the question. Provide details and share your research!

                                                              But avoid



                                                              • Asking for help, clarification, or responding to other answers.

                                                              • Making statements based on opinion; back them up with references or personal experience.


                                                              To learn more, see our tips on writing great answers.




                                                              draft saved


                                                              draft discarded














                                                              StackExchange.ready(
                                                              function () {
                                                              StackExchange.openid.initPostLogin('.new-post-login', 'https%3a%2f%2ftex.stackexchange.com%2fquestions%2f553%2fwhat-packages-do-people-load-by-default-in-latex%23new-answer', 'question_page');
                                                              }
                                                              );

                                                              Post as a guest















                                                              Required, but never shown





















































                                                              Required, but never shown














                                                              Required, but never shown












                                                              Required, but never shown







                                                              Required, but never shown

































                                                              Required, but never shown














                                                              Required, but never shown












                                                              Required, but never shown







                                                              Required, but never shown











                                                              Popular posts from this blog

                                                              Biblatex bibliography style without URLs when DOI exists (in Overleaf with Zotero bibliography)

                                                              ComboBox Display Member on multiple fields

                                                              Is it possible to collect Nectar points via Trainline?